Sie sind auf Seite 1von 153

1.

THE COMPONENTS OF A 2 DIMENSIONAL FORCE


There are two ways of getting the components of a 2 D force.
These are the use of complex number and the other is the Pol and
the Rec Commands.

Example 1 . Find the x and y components of the force shown.


y 200 N

300
x

Solution 1:
ENTER: MODE 2
INPUT: 200  30 = “  is SHIFT (-) “
DISPLAY:
173.205 + 100 i
The x component is 173.205 N and the y component is 100 N.
Solution 2:
Using the Rec Command:
ENTER: MODE 1
Input: Rec( 200,30) = “ Rec is SHIFT - and comma is SHIFT ) “
DISPLAY:
X = 173.205 Y = 100
The advantage of this technique is that the value of 173.205 is
automatically stored to X and 100 to Y.
ENTER: X “ ALPHA ) “
DISPLAY:
X
173.20Y
ENTER: Y “ ALPHA S↔D “

1
DISPLAY:
Y
100

Example 2
Find the components of the given force.
Y 400 N

4 X

Solution 1:
Using Complex Number:
ENTER: MODE 2
INPUT: 400  tan-1 ( ¾) =
DISPLAY:
320 + 240i
Fx = 320 N and Fy = 240 N
Solution 2:
ENTER MODE 1:
We will compute the angle first.
ENTER: Pol ( 4, 3) = “ Pol is SHIFT + “
DISPLAY:
r = 5 ,  = 36.8690
 is automatically stored to Y.
ENTER: Rec ( 400, Y ) =
DISPLAY:

2
X = 320 , Y = 240

Example 3 Find the components of the given set of forces shown.


(Problem taken from Engineering Mechanics by Singer 3rd ed. Page
27. )

Solution:
For angles with respect to the x axis:
1st quadrant: As is
2nd quadrant: 180 – angle
3rd quadrant: 180 + angle
4th quadrant: - angle or 360 - angle
where angle = tan-1 ( Vertical / Horizontal )
Using Complex Numbers: MODE 2
For the 600 lbs:
INPUT: 600  tan-1(3/4) =
Answer: 480 + 360i
Fx = 480 lbs Fy = 360 lbs
For the 448 lb:
ENTER: 448  ( 180 – tan-1( 2/1) ) = ( Note: 2nd quadrant)
Answer: -200.3517 + 400.703 i

3
Fx = -200.35 and Fy = 400.70
For the 260 lb :
ENTER: 260  ( 180 + tan-1( 5/12) ) = (Note: 3rd quadrant)
Answer: -240 -100i
Fx = -240 Fy = -100
For the 361 lb:
ENTER: 361  (360 – tan-1 ( 2/3) ) = (Note: 4th quadrant)
or 361  (- tan-1 (2/3) ) =
Answer: 300.37 – 200.25i
Ans. Fx = 300.37 lb and Fy = -200.25 lb

Example 4 Find the x and y components of the forces P and F


shown. Problem 2-3.5 Engineering Mechanics by Singer: 3rd ed.
page

4
Solution:
With reference to the designated x and y axes:

The slope of 400 lb is 3:4 (4th quadrant) and that of the 1200
lb is 4: 3 ( 3rd quadrant)
For the 400 lbs:
MODE 2:
Input: 400  (-tan-1( ¾) ) =
Answer: 320 – 240i
Fx = 320 lbs and Fy = -240 lbs
For the 1200 lbs:
Input: 1200  ( 180 + tan-1( 4/3) ) =
Answer: - 720 – 960i
Px = - 720 lbs Py = - 960 lbs

5
2. RESULTANT OF 2 DIMENSIONAL FORCES USING THE
METHOD OF COMPONENTS.
Example 5
Find the magnitude of the resultant and its direction using the
method of components.
Problem 2-4.2 Engineering Mechanics by Singer 3rd ed. page 27

Solution:
For the 300 lb:
Input: 300  30
Answer: 259.808 + 150i
For the 224 lbs:
Input: 224  ( 180 – tan-1(1/2) ) =
Answer: -200.352 + 100.176i
For the 200 lb:
Input: 200  ( 180 + 60)=
Answer: -100 – 173.205i
For the 390 lb:
Input: 390  ( - tan-1( 12/5) )=
Answer:

6
150 – 360i
Tabulate the result using the STAT MODE:
ENTER: MODE 3 2
INPUT
X components Y components
X Y
259.808 150
-200.352 100.176
-100 -173.205
150 -360

ENTER: AC
To get ∑Fx :
ENTER: SHIFT 1 3 2 =
Output:
∑X = 109.456
then ∑Fx = 109.456 lb
To get ∑Fy:
ENTER: SHIFT 1 3 4 =
Output:
∑Y = -283.029
Then: ∑Fy = -283.029 lb

To get the resultant and its direction:


ENTER: MODE 1
Input : Pol( 108.728, -283.029 ) =
Output:
r = 303.456  = -68.857
Resultant = 303.456 lb and  = 68.860 ( 4th quadrant )

7
Direct Solution:
A direct solution for this is using complex numbers.
ENTER: MODE 2
Input: 300  30 + 224  ( 180 – tan-1 ( ½) ) + 200  ( 180 + 60)
+ 390  ( - tan-1 ( 12/5) ) =
Output: 109.456 - 283.029i
∑Fx = 109.456 and ∑Fy = -283.029
To get the resultant:
ENTER: SHIFT 2 3 =
Output:
R = 303.456  = -68.8570

Example 6 (Past CE BOARD : Structural Engineering and Design )


For the forces in the figure shown.
a. Find the magnitude of the horizontal component of the
Resulant.
b. Find the magnitude of the Resultant.
c. Find the angle that the equilibrant makes with the axis.

Solution:
ENTER: MODE 2
To get the resultant:
Input: 86.6  30 + 70.7  (180 – 45) + 68.6  (270) =

8
Output:
25 + 24.69i
a. Ans. ∑Fx = 25 N

To get the resultant:


ENTER: SHIFT 2 3 =
Output:
35.142  44.64
b. Resultant = 35.14 N

To get the angle of the equilibrant.


Multiply by -1 . ( Equilibrant )
ENTER x -1 SHIFT 2 3 =
Display:
35.15  -135.36

Angle of the equlibrant = 135.360 ( Clockwise )

9
Example 7 ( Past CE Board )
A force of 60 kN is acting hoizontally, then another force of 50 kN
is acting at a certaing angle to the right. If the Resulant of these 2
forces is 95.4 kN.
a. At what angle does the resultant makes with the horizontal?
b. At what angle does the 50 kN makes with the horizontal?
c. Compute the horizontal component of the resultant.
Solution:

95.4 kN B

50 kN

A
60 kN C
MODE 1:
(To use the CALCULATOR Formula below:
A2  B 2  C 2
A, B and C are sides of a triangle where cos 1 ( )
2 AB
is the angle opposite side C and A and B are the other two
sides. )
A2  B 2  C 2
1
ENTER: cos ( )
2 AB

Let us compute all the angles A, B and C.


ENTER: CALC
A? 95.4 =
B? 60 =
C? 50 =
Output:

10
26.99
Therefore: Angle A = 26.990
ENTER: CALC
A? 95.4=
B? 50=
C? 60 =
Output:
32.997
Therefore: Angle B = 330
ENTER:
CALC
A? 60=
B? 50=
C? 120 =
Output:
120.012
Therefore: Angle C = 1200

We can now answer the questions.


a. The resultant makes an angle of 270 with the horizontal.
b. The 50 N force makes an angle of 180 – 120 = 600 with the
horizontal.
c. To get the horizontal component of the resultant.
Input: 95.4 cos A where A = 270
Answer: 85 kN

11
3. COMPONENTS OF A 3 DIMENSIONAL FORCE.
Example 8
A force AB of 100 N passes from A(0,0,0) to the point
B( 3, 4, 5). Find the components of the Force AB, the
direction cosines or unit vector of force AB and the angles it
makes with the coordinate axes.
Solution:
ENTER: MODE 8 1 1
Input:
[ 3 4 5 ]
ENTER: AC
To get direction cosines or unit vector:
ENTER:
Vct A ÷ Abs( Vct A )
(Note: Vct A is SHIFT 5 3
Abs(VctA) is SHIFT hyp SHIFT 5 3

Answer:
[ 0.42426 0.56568 0.70711 ]

To get the components:


Multiply the unit vector by the magnitude of the force.
ENTER: x 100
Output:
[ 42.426 55.568 70.711 ] Newtons.
Fx = 42.43 N Fy = 55.57 N and Fz = 70.71 N

12
To get the angles is makes with the coordinate axes.
x = cos-1 0.424 y = cos-1 0.566 z = cos-1 0.707
= 64.920 y = 55.530 z = 450

Example 9 PAST CE BOARD EXAM


The resultant of the concurrent forces has a magnitude of
1000 kN and acts trough the origin and the points x = 2, y =
3 and z = 4. Find the components of the force.
Solution:
ENTER:MODE 8 1 1
Input:
Vct A
[ 2 3 4]
ENTER: AC
Input 1000 Vct A ÷ Abs( Vct A ) =
( See Example 8 for the CALCULATOR KEYS )
Answer:
[ 371.39 557.08 742.78 ]
Answer: Fx = 371.39 N Fy = 557.08 N and Fz = 742.78 N

4. RESULTANT OF CONCURRENT 3 DIMENSIONAL FORCES.


Example 10 (Past CE Board Exam )
A concurrent force system in space is composed of 3 forces
descibed as follows.
P1 has a magnitude of 100 kN and acts trough the origin and
the points x = 3, y = 4 and z = 2.

13
P2 has the magnitude of 60 kN and acts trough the origin and
the points x = 4, y = 1 and z = -2 .
P3 has a magnitude of 80 kN and acts trough the origin and
the points x = 2 , y = -3 and z = 3.
Find the components of the resultant, the direction cosines of
the resultant and the magnitude of the resultant.
Solution:
ENTER: MODE 8 1 1
Input: Vct A = ( 3 4 2 )
ENTER: AC
ENTER: SHIFT 5 1 2 1
Input: Vct B = ( 4 1 - 2 )
ENTER: AC
ENTER: SHIFT 5 1 3 1
Input: Vct C = ( 2 - 3 3 )
ENTER: AC
To get the Resultant:
ENTER:
100VctA÷Abs(VctA) + 60VctB÷Abs(VctB) +
80VctC÷Abs(VctC) =
Output:
VctAns
[ 142.193 36.203 62.121 ]
Rx = 142.193 kN Ry = 36.203 kN Rz = 62.121 kN
This vector is automatically stored to VctAns

To get the direction cosines of the resultant:


ENTER: Abs ( VctAns)

14
Ouput:
Abs(VctAns)
159.3378
R = 159.34 kN

To get the unit vector or direction cosines of the Resultant:


ENTER: VctAns÷ Abs(VctAns)
Result:
[ 0.8924 0.2272 0.3898 ]

Example 11 (Problem 1.46 page 23 Pytel )


Find the components of the Force F , given that its magnitude
is 240 N.

Solution:
Coordinates of A = ( 18, 0, 0 )
Coordinates of B = ( 0, 15, 25)
Vector BA = ( 18- 0 0 – 15 0 - 25 )
= (18 -15 -25 )

15
ENTER: MODE 8 1 1
Input: VctA = ( -18 -15 -25 )
To get the components of F :
ENTER: 240VctA÷Abs(VctA) =
Output:
[ 126.08 -105.07 175.11 ]
Then Fx = 126.08 kN Fy = -105.07 kN Fz = 175.11 kN

5. RESULTANT OF NON CONCURRENT COPLANAR FORCES


Example 11 Past CE Board Exam
Consider the coplanar force system shown.

a. Find the magnitude of the resultant of the force system.


b. Compute the location of the resultant from the y axis.
c. Compute the location of the resulant from the x axis.
Solution:
To compute the magnitude of the resultant:
ENTER: MODE 2
Input: 10  90 + 6.7  tan-1 ( ½) =
Output:
5.99266 + 12. 9963i

16
Rx = 5.99 N and Ry = 12.996 N
ENTER: SHIFT 2 3 =
Output: 14.31  65.2450
Resultant = 14.31 N

b. Sum Moments at the Origin.

Components of 6.7 N
Input: 6.7  tan-1 ( ½) =
Output:
5.993 + 2.996i
Components:
5.993 Up and 2.996 to the right.
Sum Moments at Origin: (CLOCKWISE + )
∑M at O = 30 - 10(5) + 5.993(2) - 2.996(8)
= -31.982 Nm
= 31.982 Nm counterclockwise
Ry x = 31.982
12.996 x = 31.982
x = 2.461 m ( x intercept )
Rx y = 31.982
5.99 y = 31.981
y = 5.34 m ( y intercept )

17
CALCULATOR TECHNIQUE IN SUMMING MOMENTS:
To Sum Moment at O:
ENTER: MODE 3 2 SHIFT MODE ↓ 4 1
Input
X Y Freq
10 5 -1
2.996 8 -1
5.993 2 1
30 1 1

Note: X is the force, Y is the moment Arm and Freq is 1 if


clockwise and – 1 if counterclockwise.
ENTER: SHIFT 1 3 5 =
Output:
-31.982
Then Moment at 0 = 31.982 Counterclockwise

Example 12
The Force system shown consist of the Couple C and the 4 Forces.
The resultant of this system is 500 kN m counterclockwise
couple. Past CE BOARD EXAM : Problem Taken for Engineering
Mechanics by Pytel
a. What is the value of P?
b. What is the value of Q?

18
c. What is the value of C.

240

100

120

160
Solution:
(Note: All angles must be referred from the + x axis
1st quadrant: As is
2nd quadrant: 180 – angle
3rd quadrant: 180 + angle
4th quadrant: - angle or 360 – angle
where angle = tan-1 slope )

The Resultant force = 0 since Resultant is just a Couple.


P  tan-1(3/4) + 20  270 + Q  ( 180 + tan-1 ( 5/12) ) + 80  0
=0
P  tan-1(3/4) + Q  ( 180 + tan-1 ( 5/12) ) =
- ( 20  270 + 80  0 )
MODE 2:
Input: 1  tan-1 ( ¾)
Result: 4/5 + 3/5i
Input: 1  ( 180 + tan-1 ( 5/12) )
Result:

19
-12/13 – 5/13i
Input: - ( 20  270 + 80  0 )
Result:
-80 + 20i
Therefore:
P(4/5 + 3/5i) + Q(-12/13 – 5/13i ) = - 80 + 20i

Use MODE 5 1
Input:
a b c
1 4/5 -12/13 -80
2 3/5 -5/13 20

P = 200 Q = 260

Get the Components of P and Q.


for P:
Input: 200  tan-1(3/4)
Result: 160 + 120i
For Q:
Input: 260  ( 180 + tan-1 ( 5/12) )
Result: - 240 – 100i
∑M0 = 500 Nm counterclockwise
Sum Moment at O:
Assume Counterclockwise Moment Positive:
ENTER: MODE 3 2 SHIFT MODE ↓ 4 1
Input:
X Y FREQ
80 2 -1
120 6 1
20 3 -1

20
240 6 1
ENTER: AC SHIFT 1 3 5 =
Result: ∑xy = 1940

∑M0 = - C + 1940 = 500


C = 1440 kN m

6. THE DOT PRODUCT


The Dot Product or Scalar Product is :
A dot B = | A| | B| cos 
where A and B are vectors , | A| and |B| are the magnitudes
of A and B respectively and  is the angle between them.
Example 13
If A = 5i + 6j – 8k and B = 3i – 4j + 9k
Compute A dot B.
Solution:
ENTER: MODE 8 1 1
Input: VctA = ( 5 6 - 8 )
ENTER: AC SHIFT 5 1 2 1
Input: VctB = ( 3 - 4 9 )
ENTER: AC
To get the Dot Product:
ENTER: SHIFT 5 3 SHIFT 5 7 SHIFT 5 4 =
Result:
VctA dot VctB = - 81

7. ANGLE BETWEEN TWO VECTORS


Example 14
From the data of example 13, find the angle between
vector A and Vector B.
Solution:

21
Input: VctA dot VctB ÷ ( Abs(VctA) Abs(VctB) )
Then: cos-1 Ans =
Result:
134.7230
( Note: VctA = SHIFT 5 3 VctB = SHIFT 5 4
Abs(VctA) = SHIFT hyp SHIFT 5 3 )
Abs(VctB ) = SHIFT hyp SHIFT 5 4 )
8. SCALAR PROJECTION OR COMPONENT OF A VECTOR ON
ANOTHER VECTOR.
The component or Scalar Projection of Vector A on another
Vector B is :
Scalar projection of A on B = A dot B÷ Abs(B )
or Vct A dot uB
where uB is the unit vector B
= VctB÷Abs(VctB)
Example 15
Given A = 3i + 4j – 7k and B = 4i + 3j + 2k
a. Find the scalar projection of A and B.
b. Find the scalar projection of B on A.
Solution:
ENTER: MODE 8 1 1
Input: VctA = ( 3 4 - 7 )
ENTER: AC SHIFT 5 1 2 1
Input: VctB = ( 4 3 2 )
a. To find the scalar projection of A and B.
INPUT: VctA dot ( VctB ÷ Abs(VctB ) ) =
Result: 1.8569 units

22
b. To find the scalar projection of B on A.
INPUT: VctB dot ( VctA ÷ Abs(VctA) ) =
Result: 1.1625 units

Scalar Proj A
of B on A

𝜃
B

Scalar Proj of A on B

9. VECTOR PROJECTION OF A VECTOR ON ANOTHER VECTOR.


To get the vector projection of vector A on vector B:
Get the scalar projection of A on B then multiply this by the unit
vector of B.
Example 14
Given the vectors A = 3i + 4j – 6k and B = 4i – 7j + 9k.
a. Find the vector projection of A and B.
b. Find the vector projection of B on A.
Solution:
Input: VctA = ( 3 4 - 6 ) ( See Previous Examples for
Calculator Input )
VctB = ( 4 – 7 9 )
To get the vector projection of A on B.
Get the scalar projection first.

23
ENTER: VctA dot ( VctB÷Abs(VctB) )
Result: -5.79324122
Then: ENTER:
Ans VctB÷ Abs(VctB)
Result:
[ -1.9178 3.3561 -4.315 ]
This is the vector projection of A on B.

To get the vector projection of B on A.


Get the scalar projection of B on A.
ENTER: VctB dot VctA÷ Abs(VctA) =
Result: -8.962581595
Then:
ENTER: Ans Vct A÷ Abs(VctA) =
Result:
[ - 3.442 -4.59 6.8852 ]
This is the vector projection of B on A.
10. THE MOMENT OF A FORCE ABOUT A POINT.

The moment of a force about a point 0 is given by the equation:


M0  r  F
F
where r a vector from O to any point on the line
of action of vector F and F is the force vector.
Example 15

24
A force of 200 N passes from point A( 2, 3, 8) to point
B ( 4, 9, -8).
a. Find the moment vector of the 200 N force about the point C(
3, 9, 13).
b. Find the magnitude of this moment vector.
c. Find the direction cosines of this moment vector.
(PAST CE BOARD EXAM )

Solution:
Vector AB = ( 4 9 - 8 ) - ( 2 3 8 )
= ( 2 6 - 16 )
rCA = ( 2 3 8 ) - ( 3 9 13 ) = ( -1 - 6 - 5)
rCB = ( 4 9 - 8) – ( 3 9 13) = ( 1 0 -21 )

a. To get the moment vector of the 200 N about C( 3 , 9 , 13 )


Input: VctA = ( 2 6 - 16 )
VctB = ( - 1 6 - 5 )
ENTER: VctBx ( 200 VctA÷ Abs( VctA) ) =
Result: ( 1464.72 -302.24 69.748 )
This means. Mx = 1464.72 Nm
My = -302.24 Nm
Mz = 69.748 Nm
Let VctC = ( - 1 0 - 21 )
ENTER: VctC x ( 200 VctB÷ Abs( VctB) ) =
Result: ( 1464.72 -302.24 69.748 )
This is the same as the previous calculation.

b. To get the magnitude of the moment vector.


The previous calculation is stored in VctAns.
ENTER: Abs( VctAns) =
Result: 1497.2046 Nm

25
c. To get the direction cosines of the moment vector:
ENTER: VctAns÷Abs(VctAns) =
Result: [ 0.978 -0.20187 0.04659 ]

11. COUPLE VECTOR


Example 16
For the Couple shown:
a. Determine the corresponding couple vector.
b. Determine the moment of the couple about the axis GH.
(Past CE Board Exam- Example Problem 2.7 page 70 Engineering
Mechanics Statics by Pytel )

26
Solution:

4
3
4
3
4

Using the Right Hand Rule:


The Magnitude of the Couple is 100 x 0.6 = 60 kNm whose
direction is shown ( 0 3 4 )
ENTER: MODE 8 1 1
Input: VctA = ( 0 3 4 )
To get the Couple Vector:
ENTER: 60 VctA÷ Abs(VctA) =
Result: ( 0 36 48 )
The Couple Vector = 36j + 48k kNm
Store this to VctB.
ENTER: SHIFT RCL O’’’’
The Couple Vector is stored to VctB.

b. The moment of the Couple about the axis GH is just the scalar
projection of the Couple Vector about vector GH.
Coordinates:
G ( 0.6 , 0, 0 ) H( 0.3, 0, 0.3)

27
GH = ( -0.3 0 0.3 )
Input: VctC = ( -0.3 0 0.3 )
Get the Scalar Projection of VctB to VctC.
ENTER: VctB dot ( VctC÷ Abs(VctC) ) =
Result: 33.9 kNm

33.9 kNm
knkkNm

Example 17
Sample Problem 2.9 page 72 (Engineering Mechanics Statics –
Pytel )
A section of a piping system is acted on by 3 couples. Determine
the magnitude of the resultant couple vector and its direction
cosines given the magnitudes of the applied couples are:
C1 = 50 Nm C2 = 90 Nm and C3 = 140 Nm .

28
Solution:
Coordinates:
A( 0.9 , 0.3, -0.6) B( 0.7 , 0, 0 ) D( 0, -0.4, 0.5)

Following the Right Hand Rule:


Direction of C1 is from D to 0.
D0 = ( 0 0.4 -0.5 )
Direction of C2 is from 0 to B .
OB = ( 0.7 0 0 )
Direction of C3 is from A to B.
AB = ( -0.2 -0.3 0.6 )

29
Store: VctA = ( 0 0.4 -0.5 )
Store: VctB = ( 0.7 0 0)
Store: VctC = ( -0.2 -0.3 0.6 )
ENTER: 50VctA÷ Abs(VctA) + 90VctB÷Abs(VctB) +
140VctC÷Abs(VctC) =
Result: ( 50 - 28.76 80.956 ) Nm
Magnitude of Resultant Couple:
ENTER: Abs(VctAns) =
Result: 99.405 Nm
To get the Direction Cosines of the Resultant Couple Vector.
ENTER: VctAns÷ Abs(VctAns) =
Result: [ 0.503 -0.289 0.814 ]
Geometric Representation of the Answers.

30
12. RESULTANT OF NON COPLANAR PARALLEL FORCE SYSTEM
Example 18
Problem 3-37 page 114 Engineering Mechanics by Pytel
Find the resultant of the four forces acting on the plane shown.

X
Y

Solution:
R = 80k + 72k + 70k + 64k = 286k
Sum Moments about 0.
rOA = ( 6 2 0 ) rOB = ( 6 6 0 ) r0C = ( 0 6 0 )
∑M0 =
rOA X ( 0 0 70 ) + rOB X ( 0 0 64 ) + rOC X ( 0 6 0 )
Let VctA = ( 6 2 0 ) VctB = ( 0 0 70 )
rOA X ( 0 0 70 ) = ( 140 – 420 0 )
Let VctA = ( 6 6 0 ) VctB = ( 0 0 64 )
VctA x VctAB = ( 384 - 384 0 )
Let VctA = ( 0 6 0 ) VctB = ( 0 0 72 )
VctA X VctB = ( 432 0 0 )
Store: VctA = ( 140 – 420 0 )
VcTB = ( 384 - 384 0 )
VctC = ( 432 0 0 )

31
Then:
VctA + VctB + VctC = ( 956 - 804 )
Since R = ( 0 0 286 )
Then:
( x y 0 ) x ( 0 0 286) = ( 956 – 804 )
i j k
x y 0  956i  804 j
0 0 286
Then:
i( 286y) – j( 286x) = 956i – 804j
286y = 956 y = 3.342
-286x = -804 x = 2.811

The resultant is 286k passing ( 2.811, 3.342 )

CALCULATOR TECHNIQUE USING STAT MODE:


ENTER: MODE 3 2 SHIFT MODE ↓ 4 1
X Y FREQ
2 -6 70
6 -6 64
6 0 72
0 0 80

For the 70 lb , x = + 2, Y = - 6
( It will produce a + moment about x axis and – moment about y
axis using right hand rule. This is the same for all data. )

The moment of R(R = 286) with respect to the origin is


286yi - 286xk

32
To get Mx , ENTER: SHIFT 1 3 2 =
Result: ∑X = 956
To get My: ENTER: SHIFT 1 3 4 =
Result: ∑Y = -804
Then M0 = 956i – 804j = 286yi – 286xk
y = 956/286 = 3.343
x = -804/-286 = 2.811
13. REPLACING A FORCE SYSTEM BY AN EQUIVALENT FORCE
COUPLE system.
Example 19
Replace the system shown by an equivalent force couple system
with the force acting at point A, given that F = 100 lb and C = 120
lb in. Sample problem 2.11 page 81 Engineering Mechanics by
Pytel 2nd ed )

Solution:
Get Coordinates:
A( 4, 0, 2) B( 4,4,0) D( 0, 0, 2 ) E ( 0, 4, 2)

Compute F:
BE = ( -4 0 2 )
MODE 8 1 1

33
Input: VctA = ( - 4 0 2 )
Then F = 100 VctA ÷ Abs(VctA)
F = ( 89.443 0 44.721 )

For the Couple:


DB = ( 4 4 - 2 )
Store to Vct B
ENTER: SHIFT 5 1 2 1
VctB = ( 4 4 -2 )
The Couple is:
120 VctB ÷ Abs(VctB) =
C = ( 80 80 - 40 )
Sum Moment about A.
∑MA = rAB x F + C where AB = ( 0 4 -2 )
ENTER: SHIFT 5 1 3 1
Input: VctC = ( 0 4 - 2 )

Input: VctC x (100 VctA ÷ Abs(VctA) ) + 120 VctB ÷ Abs(VctB) =


Result: [ 258.89 258.89 317.77 ] kN m
ENTER: Abs(VctAns) = 484.79 kN m (Resultant Couple)
The equivalent Force System is shown.

34
14. REDUCTION TO A WRENCH ( COLLINEAR FORCE COUPLE
SYSTEM )
Example 20 Replace the three forces acting on the plate by a
wrench. Specify the magnitude of the force and the couple
moment at the point P(y, z) where its line of action intersects the
plate.

Z’

x’ y’

Solution:
Resultant Force Vector = ( -40 - 60 - 80)
Store this to VctA.
ENTER: MODE 8 1 1
Input: VctA = ( -40 - 60 - 80)

The line of action of the wrench is parallel to the line of action of


Resultant Force ( R) but not necessarily of the same sense.
The Direction Cosines of R is:
ENTER: VctA÷Abs(VctA) =

35
Result: [ -0.3714 -0.5571 -0.7428 ]

Let x’, y’, z’ be the new axes where the wrench will pass.
Then:
Mx’ = 60(12-z) + 80y
MY’ = 40Z
Mz’ = 40( 12 – y)

Assume MR = Moment of the wrench and assume that MR has the


same sense as R
Then
Mr [ -0.3714 -0.5571 -0.7428 ] =
[60( 12- z ) + 80y 40z 40(12-y) ]

This results to:


Mr ( -.3714) = 60 ( 12- z ) + 80y
MR ( -0.5571) = 40z
MR ( -0.7428) = 40( 12 – y)

This result to 3 equations 3 unknowns:


ENTER: MODE 5 2
Input:
MR y z Constant
-0.3714 -80 60 60 x 12
-0.5571 0 -40 0
-0.7428 40 0 40 x 12

Output:
MR = -623.92 ft lb
y = 0.4138 ft
z = 8. 69 ft

36
The negative sign indicates that the wrench has opposite sense
with the Resultant Force.

Alternative Solution:
R = ( -40 - 60 - 80)
Sum Moments about A( origin).
∑MA = ∑r x F
= rAB x (-60j ) + rAC x ( -40i )
= ( 0 1 2 12 ) x ( 0 - 60 0 ) + (0 12 0 ) x ( -40 0 0 )

Store: VctA = ( 0 12 0 ) VctB = ( 0 – 60 0 )


Then: VctA x VctB = ( 720 0 0 )
Store: VctA = ( 0 12 0 ) VtB = ( - 40 0 0 )
Then: VctA x VctB = ( 0 0 480 )
∑r x F = ( 720 0 0 ) + ( 0 0 480 )
= ( 720 0 480 )
This is the Couple of Resultant R = CR
whose direction is the same as that of R but not necessarily of the
same sense.
Store R to VctA = ( -40 - 60 - 80)
ENTER: VctA÷Abs(VctA) =
Result: [ -0.3714 -0.5571 -0.7428 ]
Store: Cr = [ 720 0 480 ] to Vct B
Then MR is the scalar projection of Cr to R.
ENTER: VctB dot ( VctA÷ Abs(VctA) )=
Result: -623.94

To get the value of y and z , get the components of the vector


that is perpendicular to R.

37
Cr
CN
MR

CN = CR - Mr
VctA = ( - 40 -60 - 80 )
VctB = ( 720 0 480 )
Then CN = CR - MR
= VctB - (- 623.94VctA÷ Abs(VctA)
= [ 488.2745 -347.59 16.549 ]

Then rAP x R = [ 488.2745 -347.59 16.549 ]


rAP = ( 0 y z )
( 0 y z ) x ( -40 - 60 - 80) = [ 488.2745 -347.59 16.549 ]
i j k
0 y z = [ 488.2745 -347.59 16.549 ]
 40  60  80

i( -80y + 60z ) – j( 40z) + k( 40y) = [ 488.2745 -347.59 16.549 ]


-80y + 60z = 488.2745
-40z = -347.59 z = 8.69
40y = 16.549 y = 0.4137
-80y + 60z = 488. 3 OK!

38
Example 21 (Past CE BOARD )
Given in the figure the set of vertical forces and a couple of 1200
Nm in the diagonal plane ADFG.
a. Find the x component of the resultant moment at A.
b. Find the resultant moment at A.
c. Find the direction cosines of the Resultant Moment at A.

Solution:

39
The slope of DG is 4.2 to 3.6 = 7/6
The slope perpendicular to plane ADGF is 6/7 = y/z
The Direction Number of the Couple [ 0 6 - 7]
Note: y is going up , z is going to the negative direction.
Moments of the other forces:
For 800 N:
Mx = -800 ( 3.6) = -2880 Nm
My = 0
Mz = 0
For the 100 N:
Mx = 1000(3.6) = 3600 Nm
My = 0
Mz = - 1000 x 2.5 = -2500 Nm
For 200 N:
Mx = 0
My = 0
Mz = 200 x 2.5 = 500 Nm
Moment Due to Forces:
M = ∑Mxi + ∑Myi + ∑Mz k
= ( -2880 + 3600)i + 0j + ( -2500 + 500)k
= 720i – 2000k

For Resultant Moment:


ENTER: MODE 8 1 1
Input: VctA = ( 720 0 - 2000 )
ENTER: SHIFT 5 1 2 1
Input: VctB = ( 0 6 - 7 )
To get the Resultant Moment:
ENTER:
VctA + 1200VctB÷Abs(VctB) =
Result: [ 720 780.94 -2911.108 ]

40
Then Mx = 720 My = 780.94 and Mz = -2911.108
To get the Resultant Moment:
ENTER: Abs(VctAns) =
Result: 3098.8436 Nm

Example 22
Determine the Resultant of the seven column forces that act on
the concrete slab shown. Prob 3.37 page 115 Engineering
Mechanics by Pytel SI ed

41
∑MX = R yC and ∑My = Rxc
ENTER: MODE 3 2 SHIFT MODE ↓ 4 1
Input the Given Data:
X Y Freq
0 12 40
0 0 40
-5 12 24
-6 7.5 18
-6 0 36
-9.5 7.5 20
-12 0 36

Note: If the Force Produce a negative moment about an axis using


the right hand rule, put a negative sign.
Example: For the 24 kN force:
Moment about x axis is -24(5) , put 5 as negative.
Moment about y axis is 14(12) , put 12 as is.

To get the Resultant: Just sum the Frequency.


ENTER: SHIFT 1 4 1 =
Result: n = 214
R = 214 kN downward. or R = -214k
To locate the Resultant:
ENTER: SHIFT 1 3 2 =
∑Mx = -1066
ENTER: SHIFT 1 3 4 =
∑My = 1053
Then MA = -1066i + 1053j
Moment of R with respect to Origin is
( x y 0 ) x ( 0 0 -214 )

42
i j k
x y 0  -1066i + 1053j
0 0  214
i( -214y) - j( -214x) + 0k = - 1066i + 1053j
-214y = -1066 y = 4.98 m
214x = 1053 x = 4.92 m

Example 23 - ( Sample Problem 3.5 page 101 – Engineering


Mechanics by Pytel- Kiusalas )
Determine the Resultant of the Force System shown.

Solution:
To get Resulant:
ENTER: MODE 2
Input: 120  (-40) + 60  270 + 80  30 + 50  180
Result:
111.207 – 97.1345i

43
Rx = 111.21 kN and Ry = - 97.13 kN
ENTER: SHIFT 2 3 =
R = 147. 66  -41.136

Sum Moment at 0. Assume Clockwise Moment Positive:


Components of 120 kN
Input: 120  -40 = 91.925 – 77.1345i
Components of 80 kN
Input: 80  30 = 69.282 + 40i

ENTER: MODE 3 2 SHIFT MODE ↓ 4 1


Input:
X Y FREQ
50 6 -1
69.282 6 1

44
40 8 -1
60 8 1
200 1 1
ENTER: AC
ENTER: SHIFT 1 3 5 =
Result:
∑xy = 475.692
∑M0 = 475.692 kN m clockwise.

Since the Resultant is 147. 66  -41.136 , angle is in the 4th


quadrant and the moment is clockwise , then the location of the
resultant is shown:

41.1360
d

∑M0 = Rd
475.692 = 147.66 d
d = 3.22 m

15. RESULTANT OF DISTRIBUTED LOADS


Example 24
Determine the Resultant and its Location acting on the beam
shown. Example 3.11 Engineering Mechanics by Pytel 2nd ed.
page 120

45
B
A

Solution:
Model the lines OA and AB.
ENTER: MODE 3 2
For OA
Input:
X Y
0 0
4 15
ENTER: AC
Get A: SHIFT 1 5 1 = ( A = 0)
B: SHIFT 1 5 2 = ( B = 3.75)
Equation of line OA : y = 0 + 3.75x or y = 3.75x
For AB
Input:
X Y
4 15
10 10
ENTER: AC
Get A: SHIFT 1 5 1 = (A = 55/3)
Get B: SHIFT 1 5 2 = ( B = -5/6)
Equation of line AB is y = 55/3 – 5/6x

46
To get the Resulant:
4 10
Input:  3.75x   (55 / 3  5 / 6 x)dx =
0 4
Answer: 105 kN
To get the Resultant:
4 10
Input:  3.75x( x)   (55 / 3  5 / 6 x)( x)dx =
0 4
Result: 590 (This is the moment with respect to the origin.)
Location of Resultant = 590/105 = 5.619 m

Example 25
After a severe rainstorm, the flat roof of a building is covered by
2.5 inches of rainwater. The specific weight of water is 62.4 lb/ft3,
so water at a depth of 2.5 inches causes a uniform pressure of
62.4 x 2.5/12 = 13 lb/ft2. Determine the resultant force that the
water exerts on the roof. Sample 3.12 Page 121 Engineering
Mechanics by Pytel 2nd ed. )

Solution:

47
Model the Trapezoid ABCD.
The Width W of the trapezoid is a linear function of the height x.
ENTER: MODE 3 2
Input:
X Y
0 40
75 64
ENTER: AC
Get the value of A: SHIFT 1 5 1 = ( A = 40 )
Get the value of B: SHIFT 1 5 2 = ( B = 0.32 )
Then W = 40 + 0.32x
Then Pressure at any distance x is:
x

P = 13 (40  0.32 x)dx
0
To get the magnitude of the resultant force:
75
INPUT: 13  0
(40  0.32 x)dx =
Result: 50, 700 lb
To get the location of the Resultant:
75
INPUT: 13 
0
(40  0.32 x)xdx =
Result:
2047500 (This is the moment of the Force about the reference
AB)
Then: Location of R = 2047500/50,700 = 40.38 ft

Example 26
The reaction at the base of a concrete footing may be
approximated by the piecewise linear line load as shown.
Determine the resultant force and its line of action.
Problem 3.55 page 124 Engineering Mechanics by Pytel 2nd ed.

48
Solution:
Model Line AB:
MODE 3 2
Input
X Y
0 300
1.5 400
ENTER: AC
Get A ( SHIFT 1 51 = ) ( A = 300 )
Get B ( SHIFT 1 5 2 = ) ( B = 200/3 )
Equation AB: y = 300 + 2/3x 0 ≤ x ≤ 1.5
Equation BC: y = 1.5 1.5 ≤ x ≤ 3
Model Line CD
MODE 3 2
Input:
X Y
3 400
6 0
ENTER: AC

49
Get the value of A: (SHIFT 1 5 1 = ) ( A = 800 )
Get the value of B: (SHIFT 1 5 2 = ) (B = -400/3)
Equation of line CD : y = 800 – 400/3x 3 ≤ x ≤ 6

To get the Resultant:


1.5 3 6
Input  (300  200 / 3x)  1.5dx   (800  4 / 3x)dx
0 1.5 3
Result: 1127.25 lb
R = 1127.25 lb
To get the location of the Resultant from 0.
Input:
1.5 3 6

 (300  200 / 3x) x  1.5xdx   (800  4 / 3x) xdx


0 1.5 3
Result: 2817.5625

Location of the resultant:


2817.5625 / 1127.25 = 2.4995 from 0.

16. EQUILIBRIUM OF COPLANAR FORCES


Example 27 Determine the Tensile forces in each of the chords
that support the 200 lb weight as shown.

50
Solution:
Assume all reactive forces as Tension:
For Reactive Forces all components to the right and upward
all components to the left and downward are negative.
For Applied forces:
The sign convention for reactive forces are reversed.
In this case: Reactive Forces are BC and BA and active forces is
the 200 lb.
ENTER: MODE 5 1
Input:

MODE 5 1
Input:
BC BA Applied Load (-)
cos 30 - cos 60 0 x component
sin 30 sin 60 200 = y components
Result:
BC = 100 lb BA = 173.205 lb

Example 30 The 100 lb cable is supported by two smooth surfaces


as shown. Determine the reaction on the cylinder by these
surfaces at points A and B.

51
Solution:

450 600

ENTER: MODE 5 1
Input:
A B Applied Load (-)
a b c
cos 45 -cos 60 0 x comp
sin 45 sin 60 100 = y comp

Result: A = 51.7638 lb B = 73.205 lb

Example 28
The Forces on the Gusset Plate of a joint in a bridge truss act as
shown. Determine the values of P and F to maintain equilibrium.
Problem 3-4.4 Engineering Mechanics (Statics and Dynamics by
Singer 3rd ed.

52
Solution:
Get 1st the resultant of 4000 lb and the 3000 lb forces.
ENTER: MODE 2
Input: 4000  (180 -45) + 3000  270
Result: -2828.427 – 171.5729i
ENTER: MODE 5 1
Input:
P F Applied Loads (-)
cos 15 -cos 60 2828.427 x comp
sin 15 -sin 60 171.5729 = y comp
Result: P = 3342.78 lb and F = 800.903

53
Example 29
The uniform 50 kg plank is resting on rough surfaces at A and B.
The coefficients of static friction are shown in the figure. A 100 kg
man starts walking from A towards B.
a. Compute the reaction at B.
b. Compute the reaction at A.
c. Determine the distance x when the plank will start to slide.
PAST CE BOARD EXAM

Solution:
100 Kg

50 kG

0.2NA
500

0.5NB NB

NA

54
ENTER: MODE 5 1
Input:
NA NB Applied LOAD(-)
0.2 0.5cos 40- cos 50 0 x comp
1 0.5sin40 + sin 50 150 y comp

NA = 81.644 kG NB = 62.86 kg
NA = 81.644(9.81) = 800.93 N
NB = 62.86(9.81) = 616.66 N

∑MB = 0
NA ( 4) – 100(4-x) – 50(2) = 0
Use SHIFT CALC with NA = 81.644
x = 1.73424

To compute the Reaction at A:


Input: Pol( 0.2NA, NA ) where NA = 81.644
Result: r = 83.261 kg  = 78.70
RA = 83.261 x 9.81 = 816.79 N
To Compute Reaction at B:
Input: Pol(0.5NB, NB ) =
Result: r = 70.28  = 63.430
Then: RB = 70.28 x 9.81 = 689.44 N

55
17. ANALYSIS OF TRUSSES
Example 30
Determine the force in members all members of the crane
truss shown. Problem 4-3.9 page 123 Engineering
Mechanics by Singer 3rd ed

Solution:

300

56
Find Distance AC:
Using Cosine Law Technique:
AC = | 6  120 – 9 | = 13.077
To compute angle A:
MODE 1:
A2  B 2  C 2
Input: cos-1
2 AB
A? 9 = B? 13.077 = C? 6 =
Result: 23.4110
Angle A = 23.4110

Consider Joint A:

AC

23.4110

AB

5200 lb
ENTER: MODE 5 1
Input:
AC AB LOAD(-)
-cos 23.411 -1 0
sin 23.411 0 5200 =
Result:
AC = 13,087.55 lb AB = -12,010.16 lb

57
BC

600 BA = 12,010.16
300
BD

ENTER: MODE 5 1
Input:
BC BD LOAD
-cos 60 -cos 30 12,010.16
sin 60 -sin 30 0 =

BC = -6005.08 lb BD = -10,401.104 lb

Consider triangle DBC:


To compute angle D:
ENTER: Pol( 12, 6) =
 = 26.5650 and r = 13.4164
CD = 13.4164 ft

Compute Distance DE:


ENTER: rec( 13.4164, 26.5650 + 300)

58
X = 7.392 Y = 11.196
DE = 11.196
∑MD = 0
EC(11.196) – 5200 ( 12cos 30 + 9) = 0
EC = 9006.787

Example 31 For the truss shown, determine the force in BF by the


method of joints then check using the method of sections.
Problem 4-4.4 page 130. Engineering Mechanics by Singer 3rd ed.

1200

Solution: a
By Inspection: BE = 1200 lb T and BC = 2400 lb T
Input: Pol( 9, 12) =

59
Result:  = 53.130 then Angle CAB = 53.130 = 
To get the Reactions:
Sum Moments at D.
A(18) – 2400(9) – 1200(12) = 0
A = 2000

Consider Joint A:
ENTER: MODE 5 1
Input:
AB AC Applied Loads (-)
cos 53.13 1 0
sin 53.13 0 -2000 =

Ab = -2500 lb AC = 1500 lb
Consider Joint B:
BD


2500 1200

BF
2400

ENTER: MODE 5 1
Input:
BD BF Load (-)
cos 53.13 cos 53.13 -1200 – 2500 cos 53.13
sin 53.13 - sin 53.13 2400 – 2500 sin 53.13
Result:

60
BD = -2000 lb and BF = - 2500 lb
Using Method of Section, pass a cutting plane aa . Consider the
left section.
∑MA = 0
Resolve components of BF at F.
BF sin 53.13 ( 18) + 2400(9) + 1200(12) = 0
BF = -2500 lb

18. CABLES UNDER CONCENTRATED LOAD


Example 32 (PAST CE BOARD/ SAMPLE PROBLEM 6.1 page 296
ENGINEERING MECHANICS BY PYTEL 2nd ed )
For the cable shown, determine the angles β1 and β2 and the force
in each segment , and the length of the cable.

D
C

Solution:

∑MA = 0
Then:
BC(cos35 )(6) + BC(sin 35)(24) – 2000(17)- 1600(6) = 0
Let BC = X and USE SHIFT SOLVE:

61
BC = 2333.95 lb
Consider Joint C:
CB  35 + CD  (180-β2) + 2000  270 = 0
CD  (180-β2) = - ( 2333.95  35 + 2000  270 )
MODE 2:
INPUT: - ( 2333.95  35 + 2000  270 )
SHIFT 2 3 =
CD  (180-β2) = 2023  160.920
then β2 = 19.080
Consider Joint D.
DC  (-β2) + DA ( 180 – β1 ) + 1600  270 = 0
DA (180 -β1 ) = - (DC  (-β2) + 1600  270)
where: β2 = 19.08 DC = 2023
MODE 2:
INPUT: - (DC  (-β2) + 1600  270)

Result: 2961.19  130.2140


Then: DA ( 180 – β1 ) = 2961.19  130.2140
DA = 2961.19 and β1 = 180 – 130.2140
= 49.80

To get the length of the Cable:


L = ∑l/cosβ
L = 7/cos 35 + 11/cos 19.08 + 6/cos 49.8
= 29.48 ft

62
EXAMPLE 33 (PAST CE BOARD EXAM )
From the given figure of a cable connection shown.

a. Find the Tension in segment CD.


b. Find the value of β.
c. Find the tension in segment AB.
d. Find the tension in segment BC.
d. Find the length of the cable.
Solution:

63
CD

∑MA = 0 Let CD = X
ENTER: Pol( 1.5, 2) =
Result:  = 53.130
X cos 53.13 (2) + X sin 53.13 ( 5.5) – 3( 2) – 8(4) = 0
X = 6.857 kN = CD
Consider Joint C.
6.857  53.13 + CB( 180 – β) + 8270 = 0
CB( 180 – β) = - (6.857  53.13 + 8270 )
MODE 2: INPUT - (6.857  53.13 + 8270 )
Result: 4.82172148.569
Then β = 31.431
Consider joint B.
BA(180 - ) + BC(-31.4131) + 3270 = 0
BA(180 - ) = - ( 4.82172-31.4131 + 3270)
= 6.879126.738
BA = 6.879  = 180 – 126.738 = 53.2620
Length of the Cable = ∑l/cos 
= 1.5/cos 53.13 + 2/cos 31.4131 + 2/cos 53.262
= 8.187 m

64
19. EQUILIBRIUM OF 3 D FORCES
Example 34
Find the tension in each of the 3 cables that support the 3 kip
weight shown.

Solution:
Coordinates:
A ( -3, 8, 4) B( -2, 7, -3) C( 3, 5, -3) D(0,0,0)
Consider Joint D:
DA + DB + DC + ( -3j ) = 0
Get Direction Cosines:
For DA: Input VctA = ( -3 8 4 )
For DB: Input VctB = ( -2 7 - 3 )
For DC : Input VctC = ( 3 5 - 3)
ENTER: VctA÷Abs(VctA) =
Result: [ -0.318 0.848 0.424 ] Direction Cosine of DA
ENTER: VctB÷ Abs(VctB) =
Result: ( -0.254 0.889 -0.381 ) Direction Cosines of DB
ENTER: VctC÷Abs(VctC )

65
Result: ( 0.4575 0.7625 -0.4575 ) Direction Cosines of DC
Then:
DA( -0.318 0.848 0.424 ) + DB ( -0.254 0.889 -0.381 )
+ DC(( 0.4575 0.7625 -0.4575 ) + ( 0 -3 0 ) = 0

ENTER: MODE 5 2
DA DB DC
-0.318 -0.254 0.4575 0 x comp
0.848 0.889 0.7625 3 y comp
0.424 -0.381 -0.4575 0 z comp

Note: The data are entered by colum and the load ( 0 – 3 0 )


becomes ( 0 3 0 ) since is is transferred to the right:
Answers: DA = 1.878 kips DB = 0.3135 kips DC = 1.48 kips

Alternative Solution using Matrix :


Using L = x2  y 2  z 2
DA = ( - 3 8 4 ) DB = ( -2 7 -3 ) DC = ( 3 5 -3 )
Length of DA = 89
Length of DB = √62
Length of DC = √43
We will use Matrix:
ENTER: MODE 6 1 1
Input the Direction Numbers of DA, DB and DC by Columns.
Input:
 3  2 3 

MatA = 8 7 5 

 4  3  3
ENTER: SHIFT 4 1 2 1

66
Input the lengths of DA, DB and DC as a diagonal matrix.
Input:
 89 0 0 
 
MatB =  0 62 0 
 0 0 43 

ENTER: SHIFT 4 1 3 3
Input the negative of the Applied load in a 3 x 1 matrix.
Input:
0 
 
MatC = 3
 
0 
To get the Direction Cosines of all members:
ENTER: MATA MAT B-1 ( SHIFT 4 3 SHIFT 4 4 x-1 = )
 0.317  0.254 0.4574 
Result:
 0.8479 0.889 0.7624 
 
 0.4239  0.381  0.457
This is stores in MatAns:
To get the tensile forces in all members:
ENTER: MatAns-1 MAtC ( SHIFT 4 6 x-1 SHIFT 4 5 = )
Result:
1.8784 
0.3135 kips
 
1.4797 

67
Example 35 (PAST CE BOARD EXAM )
Determine the force in each leg of the tripod shown.

Solution:
Since the legs are equally distributed in the circle, the forces on
the legs are the same. Consider only one leg.

50/3

To get  , ENTER: Pol( 1, 5)


 = 78.690

68
Then ∑Fy = 0
X sin  = 50/3
X = 16.997 lb
Example 36
Boom DC supports a 500 lb load. Determine the tension in
each cables and the pin reaction at D.

Solution:
Get Coordinates
A( 0, 6, 6) B( 0, 6, -6) C( 8, 6, 0) D(0,0,0)
Consider joint C.
CB + CA + CD + (-500j) = 0
CB = ( -8 0 6) CA = ( -8 0 -6) CD = (- 8 -6 0)
Compute lengths:
Length CB = 10
Length CA = 10
Length CD = 10
ENTER: MODE 6 1 1

69
Input Vectors CB, CA and CD by column.
 8  8  8
Input: MatA =  0 0  6
 6  6 0 
ENTER: SHIFT 4 1 2 1
Input the lengths as a diagonal matrix.
10 0 0 

MatB= 0 10 0

 
 0 0 10
ENTER: SHIFT 4 1 3 3
Input the negative of the applied load .
 0 

MatC = 500

 
 0 
To Compute the Direction Cosines of CB, CA and CD in that order:
ENTER: MatA MatB-1 =
 0.8  0.8  0.8
Result:
 0 0  0.6

 0.6  0.6 0 
This is stored in MatAns.
(Note: Direction Cosines of CB = ( -0.8 0 0.6 )
of CA = ( -0.8 0 -0.6) and of CD = ( -0.8 – 0.6 0 )
To get the loads carried by each member:
ENTER: MatAns-1 MatC =
Result:

70
 416.67 
 416.67 
 
 833.33
Then CB = 416.67 lb CA = 416.67 lb and CD = -833.33 lb
( + tension, - compression )

Then CD = 833.33 ( -0.8 -0.6 0 ) =


= - 666.64i - 500j
Then Consider Joint D.
∑F = 0
D + - 666.64i - 500j = 0
D = 666.64i + 500j ( Reaction at D)
Example 37 (PAST CE BOARD EXAM ), If the capacity of each
leg is 15 kN, what is the safe value of W?

X Y

71
Get Coordinates:
A( 0, 0, 2.4) B( 0.9, 1.8, 0) C( -1.8, 0, 0) D( 0.9, -1.8, 0)
Consider joint A:
AD + AB + AC - Wk = 0
AD= ( 0.9 -1.8 -2.4 )
AB = ( 0.9 1.8 -2.4 )
AC = ( -1.8 0 -2.4 )
Assume W = 1
Get the lengths:
AD = 3.132
AB = 3.132
AC = 3
 0.9 0.9  1.8 

Input: MatA =   1.8 1.8 0 
 2.4  2.4  2.4
3.132 0 0
Input: MatB =
 3.132 0
 0
 0 0 3
Input MatC as the negative of the applied load.
0 
 
MatC = 0
 
1 
To get the Direction Cosines of all the members:
ENTER: MatAMatB-1 =

72
 0.2873 0.2873  0.6

Result:  0.574 0.5747 0 

 0.766  0.766  0.8
To get the loads carried by the structure when W = -1k
ENTER: MatAns-1 MatC =
Result:
  0.435
  0.435
 
 0.416
This is the actual forces when W = -1k kN.
Since Capacity of any leg is 15 kn, Divide 15 by the biggest of the
values in the matrix.
W (max ) = 15/0.435 = 34.48 kN Ans.
Example 38
The light boom AB is attached to the vertical wall by the ball and
socket joint at A and is supported by two cables at B. A force P =
12i – 16k is applied at B. Note that RA , the reaction at A acts
along the boom because it is a 2 force member. Compute the
cable tensions and RA. Problem 5.22 page 230 Engineering
Mechanics Statics by Pytel.

73
Solution:
Get Coodinates:
A ( 2, 0, -3) B( 0, 6, 0) C( 2, 0, 6) D( -4, 0, 2)
Consider Joint B.
BA + BC + BD + 12i – 16k = 0
or BA + BC + BD = -12i + 16k
BA = ( 2 -6 -3) BC = ( 2 -6 6) BD = ( -4 -6 2)
Lengths: BA = 7 BC = 2√19 BD = 2√14

Input:
2 2  4

MatA =  6  6  6

 
  3 6 2 
Input the lengths as a diagonal matrix:
Input:

74
7 0 0 

MatB = 0 2 19 0 

0 0 2 14 
Input the negative of the applied load
Input:
 12

MatC = 0

 
 16 

To get the directions cosines of BA, BC and BD:


ENTER: MatAMatB-1 =
 0.2857 0.2294  0.534

Result:  0.857  0.6882  0.801

 
  0.428 0.6882 0.2673 
This is stored in MatAns
To get the forces in the members:
ENTER: MatAns-1 MatC =
Result:
 18.67 
 5.8119 
 
 14.97 

Then: BA = 18.67 kN C BC = 5.8119 kN T BD = 14.97 kN T

The reaction at A is also the same as the bar force at


BA = 18.67 kN

75
Example 39
The boom shown supports a load of W = 1400 lb. Compute the
forces in the cables AE and BD and also the components exerted
by the ball and socket joint C.

76
Solution:

Get the coordinates.


A( 8, 0, 0) B( 4, -5, 0) C( 0, -10, 0 ) D( 0, 0, -3) E( 0, 3, 6)
Sum moments from the axis CE.
To get AD, sum moment about axis CE.
CE = ( 0 13 6 )
AD = ( -8 0 -3)
W = ( 0 0 -1400)
∑Mce = ∑Mc dot unit vector of CE

∑Mc = ∑r x F = rCA X FAD + rCA X W


CA = ( 8 10 0 )
∑Mc dot unit vector of CE = 0
( 8 10 0 ) x |AD| uAD dot uCE +
( 8 10 0 ) x ( 0 -1400 0 ) dot uce = 0
where uce and uAD are unit vectors of AD.

77
For: ( 8 10 0 ) x ( 0 – 1400 0 ) dot uce
Store:
VctA = ( 8 10 0 )
VctB =( 0 -1400 0)
VctC = CE = ( 0 13 6 )
Then:
( 8 10 0 ) x ( 0 – 1400 0 ) dot uce =
ENTER: VctA X Vct B dot VctC ÷ Abs(VctC) =
Result:
-4694.4515
For :
( 8 10 0 ) x |AD| uAD dot uCE
VctA =( 8 10 0 ) =VctA
CE = (0 13 6 ) = VctC
INPUT: VctB = AD = ( -8 0 -3 )
ENTER: VctA x ( VctB÷Abs(VctB)) dot ( VctC ÷ Abs(VctC ) )=
Result:
6.4721
Then:
6.4721|AD| - 4694.4515 = 0
AD = 725.34 lb

To get BE: Sum moments about axis CD.


∑MCA = ∑MC dot uCD
= ( rCE x FBE + rCA x W) dot uCA
= ( 0 13 6 ) ( |BE| uBE dot uBE) dot uCD
+ ( 8 10 0) X( 0 -1400 0) dot uCA
For: ( 8 10 0) X( 0 -1400 0) dot uCD
VctA = ( 8 10 0 )
VctB = ( 0 -1400 0 )

78
VctC = CD = ( 0 10 -3)
ENTER: (VctA x VctB ) dot ( Vct C÷ Abs(VctC) ) =
Result: 3218.296
For:
( 0 13 6 ) ( |BE| uBE dot uBE) dot uCD
VctA = ( 0 13 6 )
VctB = BE = ( -4 8 6 )
VctC = CD = ( 0 10 -3)
ENTER: VctA x ( VctB ÷ Abs(VctB) ) dot VctC ÷ Abs(VctC) =
Result:
-3.5217
Therefore:
|BE|(-3.5217) + 3218.296 = 0
|BE | = 913.85 lb

Consider the whole structure:


C + |BE|uBE + |AD| uAD + ( 0 -1400 0 ) = 0
C = - (|BE|uBE + |AD| uAD + ( 0 -1400 0 ) )
BE = ( -4 8 6 ) = VctA
AD = ( -8 0 -3 ) = VctB
W = ( 0 -1400 0 ) = VctC
Input: - 913.85VctA ÷ Abs(VctA) - 725.34 VctB÷ Abs(VctB) – VctC
Result:
( 1018.55 721.21 -254.41 )
C = ( 1018.55 721.21 -254.41 )

79
Example 40
The boom in the figure supports a load of W = 1400 lb. Compute
the forces in cables AE and BD and also the components exerted
by the ball and socket joint C. Problem 3-6.18 page 107
Engineering Mechanics by Singer 3rd ed.

Solution:
Get the Coordinates:
A(0, 0 ,8) B( 0, -6, 4) C( 0, -12, 0) D( 5, 0, 0) E( -5, 4, 0)
To get AE:
Sum Moments about axis CE.
∑MCE = rCD x Fbd dot uCE + rCA X W dot uCE = 0
CD = ( 5 12 0 ) = VctA
BD = ( 5 6 -4 ) = VctB

80
CE = ( -5 16 0 ) = VctC
For :
rCD x Fbd dot uCE =
(VctA X |BD| VctB÷ Abs(VctB ) )dot Vct C ÷ Abs(VctC)
ENTER: (VctA X VctB÷ Abs(VctB ) ) dot Vct C ÷ Abs(VctC)=
Result: 3.807058
rCD x Fbd dot uCE = 3.807058 |BD|

For: rCA X W dot uCE


CA = ( 0 12 8 ) = VctA
W = ( 0 -1400 0 )= VctB
CE = ( -5 16 0 ) = VctC
rCA X W dot uCE =
Input: ( VctA X VctB ) dot VctC÷ Abs(VctC )=
Result: - 3340.68

Then:
3.807058 |BD| - 3340.68 = 0
|BD| = 877.5 lb

To get AE :
Sum Moment about axis CD.
∑MCD = rCA X FAE dot uCD + rCA X W dot uCD = 0
For : rCA X FAE dot uCD
rCA = ( 0 12 8 ) = VctA
FAE = |AE| uAE where AE = (-5 4 -8) = VctB
CD = ( 5 12 0 )
ENTER: (VctA x Vct B ÷ Abs(VctB)) dot VctC÷Abs(VctC) =
Result: -8.40775
rCA X FAE dot uCD = |AE|(-8.40775)

81
For rCA X W dot uCD
CA = ( 0 12 8 ) = VctA
W = ( 0 -1400 0 ) = VctB
CD = ( 5 12 0 ) = VctC
rCA X W dot uCD =
ENTER: ( VctA x VctB ) dot VctC ÷ Abs(VctC ) =
Result: 4307.692
Then:
|AE|(-8.40775) + 4307.692 = 0
|AE| = 512.35 lb

To get the reactions at C.


C + FBD + FAE + W = 0
Let BD = VctA = ( 5 6 -4 )
AE = VctB = ( -5 4 - 8)
W = ( 0 -1400 0 )
C = - |BD|uBD - |AE|uAE – W
ENTER: - 877.5 VctA ÷ Abs(VctA) - 512.35 VctB ÷ Abs(VctB) – VctC
Result: ( -250 599.996 800 )
Cx = -250 Cy = 600 Cz = 800

82
Example 41
Determine the safe vertical load W that can be supported by the
tripod shown without exceeding a compressive load of 2400 lb in
any member. Problem 3-6.10 page 105 Engineering Mechanics
by Singer 3rd ed. (PAST CE BOARD EXAM )

X
Solution:
Y
Get the Coordinates:
A ( 4, 0, 0) B( -5, 0, 0) C( 0, 6, 0) D( 0, 2, 6)
DA = ( 4 -2 -6) DB = ( -5 -2 -6) DC = ( 0 4 -6)
Get lengths of DA , DB and DC using A  B  C
2 2 2

DA = 2√14 DB = √65 DC = 2√13

83
The Direction Numbers of DA, DB and DC are written by column.
 4 5 0 

Input: MatA =  2  2 4

 
 6  6  6
Input the lengths of DA, DB and DC as a diagonal matrix.
2 14 0 0 
 
Input: MatB =  0 65 0 
 0 0 2 13 

Assume W = ( 0 0 -1 )
Input: The negative of the applied load:
0 
 
Input: MatC = 0
 
1 

To get the bar forces:


ENTER: ( MatAMatB-1)-1 MATC =
Result:
  0.46193 
 0.398136
 
  0.4006 
Divide 2400 by the biggest value in the last matrix.
W = 2400/ 0.46193 = 5,195.6 lb

84
20. ANALYSIS OF PARABOLIC CABLES
Example 42 PAST CE BOARD EXAM
The cable shown supports a horizontal load of 12.4 kN/m.
Find the tension in the cable at A and B and at the lowest
point C. Also find the length of the cable.

Solution: 12.4 x 30
53.7870 y

440

√12

-√6
85
Solution:
The equation of the parabola whose vertex is at the origin
is: y = cx2 , then √y = c1 x
This means that the square root of the vertical distance
from the origin is a linear function of the horizontal distance
from the origin.
ENTER: MODE 3 2
Input:
X Y
0 √12
30 - √6
ENTER: AC
Locate the lowest point from A.
ENTER: 0 SHIFT 1 5 4 =
0𝑥̂ = 17.5736
Get the value of A: ( SHIFT 1 5 1 = )
A = 3.4641 ( Store this to A: SHIFT RCL (-) )
Get the value of B: ( SHIFT 1 5 2 = )
B = -0.197119 ( Store this to B: SHIFT RCL 0’’’ )
The equaiton of the parabola with origin at the left.
is Y = ( A + Bx)2
Get the slope at A.
MODE 1
ENTER: d/dx(( A + Bx)2) X = 0
Result: -1.3657
Get the slope at B.
ENTER: d/dx(( A + Bx)2) X = 30

86
Result: 0.9657

Angle at A:
Input: tan-1 (1.3657) = (Disregard Sign.)
Result: 53.7870
Angle at B:
Input: tan-1 (0.9657) =
Result: 440
Using Equilibrium Technique:

ENTER: MODE 5 1
Input:
TA TB APPLIED LOAD(-)
-cos 53.787 cos 44 0
sin 53.787 sin 44 12.4 x 30
TA = 270.085 kN TB = 221.819 kN

The tension at C:
ENTER: 270.085 cos 53.787 =
159.56 kN (Horizontal Component)
or: 221.819 cos 44 = 159.56 kN

To get the length of the cable:


L
Length = 
0
1  (dy / dx) 2 dx

87
y = ( A + Bx)2
y’ = 2( A + Bx)(B)
MODE 1
Input:
30


0
1  (2( A  Bx ) B) 2 dx =

Result: 36.216 m

21. ANALYSIS OF CATENARY


Example 43
A cable is supported at A and B where B is 10 m higher than
B. The cable weighs 50 N/m and the lowest point is observed
to be 5 m lower than A and 20 m horizontally from A. Find
tehe horizontal component of the tension at any point, the
Tensions at A and B and the length of the cable.

S2
S1

Solution:
The Equation of the Catenary is:

88
y = c cosh ( x/c ) where c = H/w
At A the coordinate is ( -20, c + 5 )
Then:
c + 5 = c cosh ( -20/c )
ENTER:
c + 5 = c cosh ( -20/c ), c
ENTER: SHIFT CALC
Solve for C
This will not converge if the initial value of C is far from the
actual value.
A close approximation is to use H/w = L2/(8d) for parabolic
cable. Assume L = 2 x 40 = 80 and d = 5
Input: 802/5
Result: c = 40.807
Then: H/w = c
H/50 = 40.807
Then: H = 2040.35 N
y1 = 5 + c = 5 + 40.807 = 45.807
y2 = c + 15 = 15 + 40.807 = 55.807
T1 = wy1 = 50(45.807) = 2290.35 N
T2 = wy2 = 50(55.807) = 2790.35 N
To determine the length of the cable:
s1  s2  y1  c 2  y2  c2
2 2

= 20.81 + 38.068
= 58.878 m
Length = 58.878 m

89
Example 44
A cable weighing 20 kg/m has a span of 150 m and has a
sag of 12 m. Determine the equation of the catenary, the
tensions at the lowest and highest points in the cable and the
length of the cable.
Solution:

Solution:
The equation of the catenary is:
y = c cosh ( x/c) where c = H/w
At point B, the coordinate is ( 75, c + 12 )
ENTER: c + 12 = c cosh ( 150/c) , c
SHIFT CALC
Input: c = 1502/(8x12) (Initial Approximation )
c = 236.3483554
This is stored to variable c.
since c = H/w
H = wc
ENTER: 20c =
H = 4726.97 kg
To get tension at the highest point:
INPUT: 20( 12 + c)=

90
Result: 4966.967 kg
To get the length of the cable:
INPUT: 2 (12  c) 2  c 2 =
Result: 152.53 m
Length = 152.53 m

91
22. CENTROID OF COMPOSITE FIGURES
Example 45
A beam has a cross section shown. Locate the Centroid.
( Example 12-06 page 165 Solution to Engineering
Mechanics by Arreola SI ed )

Solution: Set the reference at the bottom.


ENTER: MODE 3 2
area distance of centroid from the reference (bottom)
X Y
200 x 50 25
30 x 210 50 + 210/2
80 x 40 50 + 210 + 40/2
ENTER: AC
The centroid is located at ∑xy / ∑x
ENTER: SHIFT 1 3 5 ÷ SHIFT 1 3 2 =

92
Result:
108.846 from the bottom.

Example 46 - From the previous example, verify also that the


moments of the areas above and below the x this axis are
equal. Example 12-07 page 165. Solution to Engineering
Mechanics by Arreola SI ed )

ENTER: MODE 3 2
Input:
above Areas distance of centroid from from gg
X Y
80 x 40 191.154 – 40/2
30 x ( 191.154 – 40) (191.154 – 40 )/2

93
ENTER: AC
∑XY = 890405.7757 (SHIFT 1 3 5 )
(Moment of above areas from centroidal axis gg )
ENTER: MODE 3 2
Input:
areas below distance of centroid from gg
X Y
50 x 200 108.846 – 50/2
30(108.846- 50) (108.846 -50)/2
ENTER: AC
∑XY = 890402.7757 (SHIFT 1 3 5 )
(Moment of areas below from
centroidal axis gg ) OK

Example 47
Locate the centroid of the shaded area shown.
Example 12-10 page 167 Solutions to Problems in
Engineering Mechanics by Arreola

94
Solution:

3 2

95
ENTER: MODE 3 2 SHIFT MODE ↓ 4 1
Input the given data:
centroid of area centrroid of area
from x axis from y axis area
X Y FREQ
6 12 24 x 12
4(12)/(3π) 24 – 4(12)/(3π) -π(12)2/4
12 + 1/3(12) 2/3(24) ½(12)(24)
ENTER: AC
To get the location of xc:
ENTER: ∑x ÷ n = ( SHIFT 1 3 2 ÷ SHIFT 1 4 1 = )
Result: 10.837
To get the location of yc:
ENTER: ∑y ÷ n = ( SHIFT 1 3 4 ÷ SHIFT 1 4 1 = )
Result: 11.3566

Location of Centroid ( 10.837, 11.3566 )

Example 48 Locate the centroid of the composite area


shown.
Sample problem 8.2 page 362 Engineering Mechanics
Statics by Pytel 2nd ed.

96
97
Solution:

2
4

ENTER: MODE 3 2 SHIFT MODE ↓ 4 1


Input:
X Y FREQ
200 700/2 700 X 400
400 – 1/3(200) 700- 1/3(400) - ½(400)(200)
-200 700/2 400 x 700
-( 400 – 4(300)/(3π)) 400 -π(300)2/2

To get xc:
Input: ∑x ÷ n = ( SHIFT 1 3 2 ÷ SHIFT 1 4 1 = )
Result: 66.5965 mm
To get yc: INPUT: ∑y ÷ n = ( SHIFT 1 3 4 ÷ SHIFT 1 4 1 =)
Result: 308.44
The centroid is located at ( 66.6, 308.44 ) mm

98
Example 49
Calculate the centroidal coordinates of the shaded surface
shown.

Solution:

4
3
2

99
ENTER: MODE 3 2 SHIFT MODE ↓ 4 1
Input:

X Y AREA
1/3(4) 1/3(6) ½(6)(4)
4(4)/(3π) 0 π(4)2/4
0 3 6x4
0 3 -π(1)2

To get xc : Input ∑x ÷ 𝑛 ( SHIFT 1 3 2 ÷ SHIFT 1 4 1 = )


Result: 0.822 in
To get yc: INPUT: ∑y ÷ n = ( SHIFT 1 3 4 ÷ SHIFT 1 4 1 =)
Result: 1.906 in

We have to modify x
Edit the data for x: This will represent the data for z:

X Y AREA
0 1/3(6) ½(6)(4)
4(4)/(3π) 0 π(4)2/4
2 3 6x4
2 3 -π(1)2
ENTER: AC
Input ∑x ÷ 𝑛 ( SHIFT 1 3 2 ÷ SHIFT 1 4 1 = )
Result:
1.388 in

The location of the centroid is at:

100
(0.822, 1.906, 1.388 ) in

23. MOMENT OF INERTIA OF COMPOSITE FIGURES


Example 50 Find the centroidal moment of Inertia Igg of the
T section shown. Example 13-11 Page 196 Solution to
Engineering Mechanics by Arreola SI ed )

Solution:
ENTER: MODE 3 2 SHIFT MODE ↓ 4 1
Input the given data:
We will locate the centroid 1st from the x axis.
Stand By distance from x axis AREA
X Y FREQ
- 30 60 x 200
- 60 + 250/2 40 x 250
ENTER: AC
To get yc : Input ∑y ÷ n ( SHIFT 1 3 4 ÷ SHIFT 1 4 1 =)
Display:
100.4545 (Store this to variable X )
ENTER: SHIFT RCL )

101
Then: EDIT the DATA.
ENTER: SHIFT 1 2
ENTER:
X Y FREQ
30-A 30 60 x 200
185-A 60 + 250/2 40 x 250
ENTER: AC

INPUT:
40 x 2503/12 + 200 x 603/12 + ∑x2 =
( ∑x2 is SHIFT 1 3 1 )
Result: 186.7287879 x 106 mm4
IGG = 186.7287879 x 106 mm4

102
Example 51
Sample Problem 13-14 page 198 Solution to Engineering
Mechanics by Arreola SI ed ) Find IGG of the section shown.

Solution:
ENTER: MODE 3 2 SHIFT MODE ↓ 4 1
Input:
STAND BY location of yc Area
X Y FREQ
25 50 x 200
50 + 210/2 210 x 30
50 + 210 + 40/2 80 x 40
ENTER: AC
To get yc : ENTER ∑y ÷n = ( SHIFT 1 3 4 ÷ SHIFT 1 4 1 = )
Result: 108.846
Store this to A: ENTER: SHIFT RCL (-)
Input the value of X for STAND BY

103
STAND BY location of yc Area
X Y FREQ
25-A 25 50 x 200
155-A 155 210 x 30
280-A 280 80 x 40
ENTER: AC
To get IX (centroidal moment of Inertia )
INPUT: 80 x 403/12 + 30 x 2103/12 + 200 x 503/12 + ∑X2
(∑X2 is SHIFT 1 3 1 = )
Result: 203.124 x 106 mm4
CENTROIDAL I parallel to the x axis is 203.124 x 106 mm4.

24. SOLVING FRICTION PROBLEMS


Example 52
A 200 lb block is in contact with a plane inclined at 300 with
the horizontal. A force P parallel to and acting up the plane is
applied to the body. If the coefficient of static friction is 0.2,
Example problem 5-4.1 Engineering Mechanics by Singer 3rd
ed.
a. Find the value of P to just cause the motion to impend up
the plane.
b. Find the value of P to just prevent motion down the plane.
Solution: a.Impending Motion up the plane.
200
y 300
x

0.2N
P
300 N 104
ENTER: MODE 5 1
Input:
P N load (negative of actual direction)
1 -0.2 200 sin 30 x component
0 1 200 cos 30 y component
ENTER =
Result: P = 134.64 N = 173.2

b. Impending Motion below the plane.


The directon of 0.2N will just be upward.
ENTER: MODE 5 1
Input:
P N load ( negative of actual direction)
1 0.2 200 sin 30
0 1 200 cos 30
ENTER: =
Result: P = 65.35 and N = 173.2

Example 53
A 200 lb block is at rest on a 300 incline. The coefficient of
friction between the block and the incline is 0.2. Compute
the value of a horizontal force P that will cause the motion to
impend up the incline. Example 5-4.2 Engineering Mechanics
by Singer 3rd ed.

105
Solution: Motion is Impending Up the Plane.

Y 200
X
300

0.2N
P
N

300

ENTER: MODE 5 1
Input:
P N Load (negative of actual direction)
cos 30 -0.2 200sin 30
-sin 30 1 200 cos 30
ENTER: =
Result: P = 175.766 lb and N = 261.09 lb

Example 54
Determine the force P required to move the wedge shown.
The angle of friction for all surfaces in contact is 150.

106
Solution:

0.268N2

N2

0.268N1
150
N3 0.268N3
N1

107
Consider the 1st wedge.
ENTER: MODE 5 1
Input:
N1 N2 Applied Loads ( - )
-0.268 1 500
1 -0.268 2000 =

N1 = 2299.13 N2 = 1116.17 lb

Consider the 2nd wedge :


ENTER: MODE 5 1
Input:
P N3 Applied Loads (-)
0 1cos 15 -0.268sin 15
0 0 1116.17 x comp
-1 1sin 15 +0.268 cos 15 -0.268(1116.17) y comp
0 0

Result: P = 943.625 lb and N3 = 1244.944 lb

25. MOHR’S CIRCLE CALCULATOR TECHNIQUE IN SOLVING


MAXIMUM AND MINIMUM MOMENTS OF INERTIA

Example 55
A certain area has the following properties:
Ix = 400 mm4 Iy = 1,000 mm4 Pxy = 400 mm4
Determine the maximum and minimum moment of Inertia.
ENTER: MODE 2
Store 400 + 400i to A (Ix , Pxy )

108
and 1000 – 400i to B. (Iy, -Pxy )
Get the Center of the Circle:
A B
ENTER : 
2
Result: 700 (700,0 )
Store this to C: This is the center of the circle.
To get the radius of the circle:
ENTER: |A-C|
Result: 500
Store this to D.
The maximum I is:
ENTER: C + D =
Result: 1200 mm4
The minimum I is :
ENTER: C – D =
Result: 200 mm4

Example
A certain Area has the following values with respect to x and y
axes.
Ix = 200 x 104 mm4 Iy = 80 x 104 mm4 and Pxy = 30 x 104
mm4
Find the maximum and minimum moment of Inertia.
Solution:
We will modify the solution given in the last problem.
ENTER: MODE 2
A B
Input: C  : D | A  C |: C  D : C  D
2
Code:
locate center: compute radius : compute max I : compute min I

109
ENTER: CALC
A? 200 x 104 + 30 x 104 i (Input Ix, Pxy )
B? 80 x 104 - 30 x 104i (Input Iy , -Pxy)
Result:
C = 1400000
D = | A – C|
670820.3932
C+ D
2070820.393 (I max )
C–D
729179.6068 (I min )

Example 56
For the region shown
a. Calculate the centroidal principal moments of Inertia and
principal directions.
b. Compute the moments and product of inertia about the UV axes
through the centroid D.
Sample problem 9.10 page 455 Engineering Mechanics (Statics
and Dynamics Pytel Si Ed)

110
Solution:
Locate the centroid.
ENTER: MODE 3 2 SHIFT MODE ↓ 4 1

Locate C.
Input the given data.
xc yc Area
X Y FREQ
80 15 30 x 160
15 30 + 170/2 =( 115) 170 x 30

To get xc: ENTER: ∑x ÷ n =


Result: 46.515
To get yc: ENTER: ∑y ÷ n =
Result: 66.515
C is at ( 46.515, 66.515 )
To get centrodal Ix and Iy

111
MODE 3 2
Input:
transferred x dist transferred y dist area
X Y FREQ
80 – 46.515 15 – 66.515 30 x 160
15 – 46.515 115 – 66.515 170 x 30

To get centroidal Ix
ENTER: 160 x 303/12 + 30 x 1703/12 + ∑Y2
Result: 37369772.73 ( Store this to E )
To get the Centroidal Iy:
ENTER: 30 X 1603/12 + 170 x 303/12 + ∑x2
Result: 21069772.73 (Store this to F )

To get the centroidal product of Inertia.


ENTER: ∑xy =
Result: - 16072727.27 (Store this to M )

Ixc = E and IyC = F and Ixcyc = M

MODE 2:
A B
INPUT: Input: C  : D | A  C |: C  D : C  D
2
Code:
locate center: compute radius : compute max I : compute min I
ENTER: CALC
A? E + Mi (Input Ix, Ixy )
B? F - Mi (Input Iy, -Ixy )
Result:
C = 29219772.72 Center is at ( 29219772.73, 0 )

112
D = | A – C | = 18020961.74 (Radius of the Circle)
C + D = 47240734.47 mm4 ( I max )
C – D = 11198810.99 mm4 ( I min )

To get Iu and Iv (angle is rotated 500 which will be rotated


1000 from the circle)
ENTER:
C + D  ( ( arg( A –C ) + 100 ) ) =
Result:
43633086.51 + 1081718182.99i
Then Iu = 43633086.51 mm4
Iuv = 1081718182.99i

26. BELT FRICTION


Example 57 A body weighing 500 N is held by a rope that
passes over a horizontal drum, the angle of contact being
1500. If the coefficient of friction between the rope and the
drum is 0.3,
a. Find the least force that will raise the body.
b. Find the least force that will hold the body.
Problem 11-17 page 137 Solution to Problems in
ENGINEERING MECHANICS SI ed Arreola
Solution:

1500

113
500 N
Solution:
Since T1 = T2eβ T1 > T2  = coeff of friction
β = angle
ENTER: MODE 3 5
Input:
angle Force (T1 )
X Y
0 1 (If  = 0, assume T1 = 1 )
180 e π(0.3) ( If  = 1800 = π , T1 = eπ(0.3) )

a. To get the least force to raise the block:


ENTER: 150𝑌̂ ( 500) = ( 150 SHIFT 1 5 5 = x 500)
Result: 1096.64 N

b. To hold the body:


ENTER: 500÷ 150𝑌̂ = ( 500 ÷ 150 SHIFT 1 5 5 = )
Result: 227.97 N

Example 58
Find the minimum number of turns the rope should be
wound around the cylinder to maintain equilibrium of the
weights shown. The coefficient of friction between the rope
and the cylinder is 0.2. Example 11-16 page 137 (US

114
Engieer Training Exam ) Solutions to Problems in Engineering
Mechanics by Arreola.

Solution:

T1
T2

ENTER: MODE 3 5
Input:
Angle T1 (Force)

115
X Y
0 1
180 eπ(0.2)

If T1 =1 then T2 = 80/2 = 40
We are looking for the angle.
ENTER: 40 𝑋̂ = ( 40 SHIFT 1 5 4 = )
Result: 1056.7860
To get the number of turns, divide this by 360.
Result: 2.9355 turns.

27. Solving Reactions with Distributed Loads


Example 59
A reinforced concrete slab in the shape of an isoceles
tapezoid weighs 3,600 N/m2. It is supported at the 3 points
A, B and C as shown. Determine the reactions at these
points. PAST CE EXAM

116
Y

1m Y

X X

Locate the centroid from the Y axis.


The width y is a linear function of distance x .
ENTER MODE 3 2
Input
X Y
0 2
5.4 2.4
Then A = 2 ( SHIFT 1 5 1 = )
B= 2/27 ( SHIFT 1 5 2 S ↔ D )
To find the Area:
MODE 1
5.4
Input:  (2  2 / 27 x)dx =
0

Result: 11.88

117
To locate the centroid:
5.4
Input :  (2  2 / 27 x) xdx
0
Result: 33.048
Lo locate the centroid.
Divide 33.048÷11.88
Result: 2. 7818

The centroid C is at ( 2.7818, 0)


Support A is at ( 4.1, -0.95)
Support B is at ( 4.1, 0.95)
Support C is at (0.5, 0.95)
Total Weight = 3600 x 11.88 = 42,768 N
∑MAB = 0
-42,768( 4.1 – 2.7818) + RC ( 4.1 – 0.5) = 0
Rc = 15,660.216 N

To get the reaction at A:


∑MCB = 0
-42,768( 0.95 – 0) + RA ( 1.9) = 0
RA = 21,384 N

To get the reaction at B.


∑MAC = 0
The centroid C is at ( 2.7818, 0)
Support A is at ( 4.1, -0.95)
Support B is at ( 4.1, 0.95)
Support C is at (0.5, 0.95)

Get the equation of line AC

118
MODE 5 1
INPUT
X Y 1
4.1 -0.95 1
0.5 0.95 1
Result: 0.4347x + 0.8238y – 1 = 0
Distance fro support B to the line:
Ax  BY  C
ENTER:
A2  B 2
A? 0.4348
X? 4.1
B? 0.8238
Y? 0.95
C? -1
Result: 1.6804
Get the distance from Centroid to the line:
All data are the same except x = 2.7818 and y = 0.
Result: 0.2246
Then ∑MAC = 0
-42,768 (0.225) + RB ( 1.6804) = 0
RB = 5726.5 N

28. Solving Newton’s 2nd Law Applications


Example 60 (PAST CE BOARD EXAM )
In the figure, the coefficient of friction under each block is
0.2. Determine the acceration of the system, the tension in
the cord connecting the blocks, and the distance traveled by

119
each block to attain a velocity of 3 m/s. Assume the pulley to
be weightless and frictionless.
coefficient of friction for all contact surfaces = 0.2 .

890 N a

1780 N T B m2a

a T F2
N2
A F1

m1a
300 N1

Solution:
The direction of motion is evidently A downward and B to
the left. Corresponding F1 = ma are applied.
ENTER: Rec( 1780, 30)
Result: ( 1541.53 , 890 )
Then N1 = 1541.52 and Component of 1780 parallel to the
incline is 890 N
F1 = 0.2 ( 1541.53 ) = 308.306
F2 = 890(0.2) = 178
A summation of forces parallel to the incline for A = 0
and a summation of forces along the horizontal for P can be
directly solved using MODE 5 1.

120
ENTER: MODE 5 1
Input:
T a Reversed Applied Loads
1 1780/9.81 890 - 308.306
-1 890/9.81 - 178

Result: T = 312.564 N and a = 1.483 m/s2


Form v2 = v02 + 2as
32 = 02 + 2(1.483) s
s = 3.04 m.

Example 61 Past CE Board Exam


Determime the Force P that will give the block shown an
acceleration of 2.5 m/s2. The coefficient of Kinetic Friction is
0.26.

W= 200 N 5

12

121
80 N
Solution:
P

ma 

0.26N

To get , ENTER: Pol( 12,5)


 = 22.620
ENTER: MODE 5 1
Input:
P N APPLIED LOAD
cos 22.632 -0.26 80/9.81 (2.5)
sin 22.62 1 80
Result:
P = 40.26 N and N = 64.52 n

122
Example 61 (PAST CE BOARD EXAM )
Two 450 N blocks are connected by a light flexible cord
passing over a frictionless pulleys shown. If the coefficient of
friction under the blocks is 0.15, find the tension in the cord
and the acceleration of the system.

mAa T
a a

mBa

FB
NA
FA NB

Solution: The direction of motion is evidently A going down


and B going up.
ENTER: Rec( 450, 60)
Result: ( 225, 389.711)
Then: NA =225 and W parallel to inline = 389.711
Then FA = 0.15(225) = 33.75 N
ENTER: Rec( 540, 30)
Result: ( 389.711, 225 )
Then NB = 389.711
W parallel to the incline = 225

123
FB = 0.15 X 389.711 = 58.457
Sum forces along the incline for both A and B.
ENTER: MODE 5 1
Input:
T a Reverse Applied Load
1 450/9.81 389.711 – 33.75
-1 450/9.81 -225 - 58.457

Result: T = 319.709 N and a = 0.79 m/s2

Example 61 (PAST CE BOARD EXAM )


In the figure shown, the cofficient of kinetic friction under
block A is 0.2 and under block B is 0.3. Determine the
tension in each cord and the acceleration of the system.

Solution:
It is evident that the 800 N block will go down and both the
200 N and the 400 N block will go up.

124
mBa T2

T1
mAa
F2

N2
F1 mC a
N1

Let us do some preparations.


The unknonws are T1, T2 and a.
We need 3 equations in 3 unknowns.
Pol( 4, 3) =
Result:  = 36.870 ( Angle of the incline with respect to the
horizontal )
ENTER: Rec( 200, 36.87)
Result: ( 160, 120 )
N1 = 160 F1 = 0.2 x 160 = 32
W parallel to the incline = 120
ENTER: Rec( 400, 36.87 )
Result: ( 320, 240 )
N2 = 320 F2 = 0.3 x 320 = 96
W parallel to the incline = 240 N

125
ENTER: MODE 5 2
Input:
T1 T2 a Reverse Applied Loads
1 0 -200/9.81 120 + 32
-1 1 -400/9.81 240 + 96
0 1 800/9.81 800
Result:
T1 = 196.57 N T2 = 621.71 N a = 2.186 m/s2

Example 62
The pulleys shown are weightless and frictionless.
Determine the acceleration of the body B and the tension in
the cable. PAST CE BOARD EXAM

126
Solution:
Assume that the 400 N block will go down.

mA(2a)

mBa

ENTER: MODE 5 1
Input:
T a Reversed Applied Loads
2 400/9.81 400
1 - 300/9.81 (2) 300
Result: T = 225 N and a = -1.226 m/s2.
Assumption of direction is wrong. The 40O N block goes up
and the 300 N block goes down.

127
29. Solving Momentum Problems
Example 64
In the figure, the 50 N ball and a 40 N ball moving as
indicated meet in oblique impact. If e = 0.6, determine the
amount and direction of the velocities after impact. e = 0.6.
Assume the surfaces of the balls are smooth.

Solution:
MODE 5 1
Input:
mass 1 mass 2 Initial Momentum of both blocks
-1 1 e ( relative vel of the blocks )

Input:
50 40 50(10cos30) + 40(-15cos 45)
-1 1 0.6( 10cos 30 - ( -15 cos 45 ) )
Then v1 = -5.0406 m/s v2 = 6.519 m/s

Direction of the 50 N block:


ENTER: Pol( -5.0406, 10sin 30)
v = 7.104 m/s at  = 135.260 ( from x axis )
Direction of the 40 N block:

128
ENTER: Pol( 6.519, -15cos 45 )
v = 12.45 m/s  = -58.420 ( 4th quad )

Example 65 The bodies shown are moving to the right with


the velocities indicated. If e = 0.75, determine the velocity of
each body after impact.

Solution:
MODE 5 1
Input:
mass 1 mass 2 Initial Momentum of both blocks
-1 1 e ( relative vel of the blocks )

MODE 5 1
Input:
10 20 10(10) + 20(4)
-1 1 0.75( 10 -4) )
Result: v1 = 3 m/s and v2 = 7.5 m/s

129
Example 66
The bodies shown move toward each other with the velocities
indicated. If the coefficient of restitution e = 0.75, determine
the resulting velocity of each body after impact.

Solution:

Solution:
MODE 5 1
Input:
mass 1 mass 2 Initial Momentum of both blocks
-1 1 e ( relative vel of the blocks )

MODE 5 1
Input:
10 20 10(10) + 20(-4)
-1 1 0.75( 10 – (-4) )
Result:
V1 = -6.33 m/s and v2 = 4.67 m/s

130
30. Normal and Tangential Component of Acceleration
Example 67 (Sample problem 9-7.1 page 365
Engineering Mechanics by Singer 3rd ed )
A particle moves in the xy plane with ax = - 6 fps2 and
ay = - 30 fps2. If the initial velocity is 100 fps dtected at a
slope of 4 is to 3 . Compute the radius of curvature of the
path 2 sec later.
Solution:
an = v2/ρ
a = ( - 6 - 30 )
an

at
100 fps

4
3

Vx = 100(3/5) = 60
Vy = 100 ( 4/5) = 80
Velocity vector when t = 0.
V = ( 60 80 )
a = ( -6 - 30 )
Since v = v0 + at
Then velocity after 2 sec = ( 60 80 ) + 2 ( - 6 - 30)
Store VctA = ( 60 80 )
Store VctB = ( -6 - 30 )

131
Then: VctA + 2VctB = ( 48 20 ) ( Store this to VctC)
To compute the tangential acceleration vector:
The direction of the acceleration vector after 2 sec is also
the same as the direction of the velocity vector.
So the tangential acceleration vector is the vector projection
of VctB to VctC.
ENTER: ( VctB dot VctC÷Abs(VctC) )
Result: -17.0769 (This is stored to Ans )
(Magnitude of the tangential acceleration)
To compute the vector projection:
ENTER: AnsVctC÷Abs(VctC)
Result : ( -15.763 -6.568 )
This is stored to VctAns
Since a = an + at
Then VctB = an + VctAns
Then: an = VctB - VctAns
ENTER: VctB – VctAns
Result: ( 9.7633 -23.432 ) This is the normal acceleration.
We have to get the magnitude of the Normal Acceleration:
ENTER: Abs(VctAns) =
Result: 25.3846
Then an = 25.3846
and the magnitude of velocity after 2 sec.
ENTER: Abs(VctC) =
Result: 52
Then an = v2/ρ
25.3846 = 522/ρ
ρ = 106.52 ft

132
Summary of Results:
Velocity after 2 sec = VctC = ( 48 20 )
at = ( -15.763 -6.568 )
an = ( 9.7633 -23.432 )
acceleration vector = VctB = ( -6 - 30 )

Alternative Solution:
v x a = v x ( an + at ) = v x an since v x at = 0 collinear)
v x a= v an sin 900
| v x a | = v(v2)/ρ
The radius of curvature can be found by the formula:
v3

| va |
From our calculations:
v after 2 sec = ( 48 20 ) Store to VctA
a = ( -6 - 30 ) Store to VctB
ENTER: Abs( VcA)3 ÷ Abs( VctA x VctB )
Result: 106.52

Example 68 Sample 9-7.2 page 366 ENGINEERING


MECHANICS by Singer 3rd ed.
The motion of a particle is defined by the position vector
r = 5ti + 3t2j + 1/3t3k where r is in ft and t is in seconds. At
the instant when t = 2sec, find the tangential and normal
components of acceleration and the principal radius of
curvature.
Solution:
r = 5ti + 3t2j + 1/3t3k
dr/dt = ( 5 6t t2)

133
= ( 5 12 4 ) (when t = 2) = velocity
d2r /dt2 = ( 0 6 2t )
= ( 0 6 4 ) ( when t= 2 ) = acceleration
Store: VctA = ( 5 12 4)
VctB = ( 0 6 4)
The tangential acceration is the vector projection of VctB to
VctA since tangential acceleration is tangent to the path.
ENTER: VctB dot VctA ÷ Abs(VctA) =
Result: 6.47 ( Magnitude of the tangential acceleration)
Tangential Acceleration:
ENTER: Ans VctA÷ Abs(VctA) =
Result: ( 2.378 5.7081 1.9027 )
Store this to VctC.
Since at + an = a
Then: an = a - at
= VctB – VctC
ENTER: VctB – VctC
Result: ( -2.378 0.2918 2.0972 ) = Nornal Acceleration:
ENTER: Abs(VctAns) =
Result: 3.1844 (Magnitude of an )
ENTER: Abs(VctA) =
Result: 13.6 magnitude of velocity
Then an = v2/ρ
3.1844 = 13.62/ρ
ρ = 58.096 ft

Alterantive Solution:
The radius of curvature can be found by the formula:

134
v3

| va |
v = VctA = ( 5 12 4)
a = ( 0 6 4) = VctB
ENTER: Abs(VctA)3÷ Abs( VcA x VctB )=
Result: 58.0953 ft

31. Radial and Transverse Components of Acceleration


(Cylindrical Coordinates)
Working Formulas:
vr = dr/dt
v = r d/dt
ar = r’’ - r (d/dt)2
a = r’’ + 2r’’
Example 68 Sample Problem 9-8.1 Engineering
Mechanics by Singer 3rd ed.
The plane curvilinear motion of a particle is defined as
r = t3/3 + 2t and  = 0.3t2 where r is in inches and  is in
radians and t is in seconds. At the instant when t = 2 sec,
determine the magnitudes of velocity, acceleration and the
radius of curvature of the path.
Solution:
r = t3/3 + 2t
ENTER: X3/3 + 2X
CALC X? 2 =
Result: 20/3 r = 20/3 when t = 2
ENTER: d/dx (X3/3 + 2X )X =2

135
Result: 6 r’ = 6 when t = 2
ENTER: d/dx ( 3(x2)/3 + 2 )X =2
Result: 4 r’’ = 4 when t = 2
For  = 0.3t 2

ENTER: 0.3X2 CALC X? 2 =


Result: 6/5  = 6/5 when t = 2
ENTER: d/dx ( 0.3X ) x = 2
2

Result: 6/5 ’ = 6/5 when t = 2


ENTER: d/dx( 0.3(2x) ) x = 2
Result: 3/5 ’’ = 3/5 when t = 2

Summary:
r= 20/3 (A)  = 6/5 (D)
r’ = 6 (B) ’ = 6/5 (E)
r’’ = 4 (C) ’’ = 3/5 (F)

ar = r’’ - r ’ 2 r’ = dr/dt
a = r’’ + 2r’’ vr = r’

To Compute the radial and transverse acceleration:


ENTER: C – AE2: AF + 2BE
CALC
C? 4 = A? = 20/3 = E? 6/5 = F? 3/5 = B? 6
Result: C-AE2 = -28/5 = -5.6
and AF + 2BE = 92/5 = 18.4
ar = -5.6 in /s2 and a = 18.4 in /s2
vr = 6 and v = r’ = 20/3( 6/5) = 8
velocity vector = ( 6 8 )

136
acceleration vector = ( -5.6 18.4 )
To get the radius of curvature:
USE: ρ = v3 /| v x a |
Store: VctA = ( 6 8 )
VctB = (( -5.6 18.4 )
ENTER: Abs(VctA)3 ÷ Abs ( VctA x Vct B ) =
Result: 6.443
Radius of Curvature = 6.443 in
Magnitude of Velocity:
ENTER: Abs(VctA) = 10 in/s
Magnitude of Acceleration:
ENTER: Abs(VctB) = 19.233 in/s2
To get the tangential acceleration:
Get the vector projection of acceleration vector to the
velocity vector.
ENTER: VctB dot Vct A ÷ Abs(VctA ) =
Result: 11.36 (This is the scalar projection)
ENTER: Ans VctA ÷ Abs(VctA ) =
Result: ( 6.816 9.088 ) This is the tangential acceleration.
Store this to Vector C.
Since a = at + an
Then: an = a - at
ENTER: VctB - VctC =
Result: ( -12.416 9.312 )
This is the Normal Acceleration:
To get the magnitude of the normal acceleration.
ENTER: Abs(VctANS)=
Result: 15.52

137
Since an = v2/ρ
Then: 15.52 = 102/ρ
ρ = 6.443 in

32. Plane Motion Analysis


Working Equations:
SB = SA + SB/A where SB/A = r
vB = vA + vB/A where vB/A = r
and aB = aA + aB/A where aB/A = r2 + r

Example 69 Sample Problem 12-4.1 page 438


Engineering Mechanics by Singer 3rd ed.
The wheel of radius 3 ft shown rolls freely to the right. At the
given position,  = 3 rad/s and  = 5 rad/s2 both
clockwise. Compute the acceleration and velocity at the
point on the wheel which is 2 ft from A.

138
Solution:
For velocity at B.

600

VB/A

vA

vB = vA + vB/A
|vA |= r = 3(3) = 9 ft/s
|v B/A | = R = 3(2) = 6 ft/s
Then vB = 90 + 6(-60)
USE MODE 2:
Input: 90 + 6(-60)
Result: 13.077(-23.41o)
VB = 19.3077 ft/s acting at 23.410 from the x axis.

139
For acceleration at B.
300

an = w2r = 2(3)2 rad/s2 = 18 rad/s2

600
aA = r at = r
3(5)= 5(2) = 10 rad/s2
15 r/s2

aB = aA + aB/A
aB = 150 + 10(-60) + 18( 210)
ENTER MODE 2:
Input:
150 + 10(-60) + 18( 210)
Result:
18.2(-75.970)

aB = 18.2 m/s2 and  = 75.970 from x axis.

140
33. DYNAMIC ACTIONS OF JET STREAMS
Example 70
A jet of water 2 in in diameter issues from a nozzle with a
velocity of 100 ft/s and impinges tangentially upon a perfect
smooth stationary vane which deflects trough an angle of 300
wiothout loss of velocity. What is the total force exerted by
the jet on the vane? Example 1518 page 435 Engineering
Mechanics by Singer 2nd ed.

Fx
Fy
Solution:
Use the Formula:
F = ρQ (Vin - Vout)
where Q = Discharge in ft3/s or m3/s
ρ = masss density in slugs /ft3 = 1.938 slugs/ft3
or in kg/m3 ( 1000 kg/m3)
Vin = velocity coming in ( ft/s or m/s)
Vout = velocity coming out ( ft/s or m/s )
where Q = Area x Velocity
F = Force exerted by the jet on the vane.

141
Compute Q:
Q = π(2/12)2/4 ( 100) = 2.18166 ft3/s
Then F = 1.938 ( 2.1866) ( 1000 - 10030 )
USE MODE 2:
Result: 56.77 - 211.88i
Fx = 56.77 lb ( to the right )
Fy = 211.88 lb ( down )

Example 71 Past CE BOARD


A jet having a diameter of 50 mm and a velocity of 15 m/s is
deflected by a vane which is curved through an angle of 600
and which is moving with a velocity of 6 m/s in the same
direction as the jet. Determine the x and y components of
the force exerted by the jet and its direction and velocity as it
leaves the vane.

142
Solution:
F = ρQ( Vin – Vout ) where Q = Av

|Vin| = 15 – 6 = 9 m/s
|Vout |= 9 m/s
F = 1000 ( π(0.05)2/4 ) (9) ( 90 - 960 )
USE MODE 2:
Result: 79.52 – 137.735i
Then: Fx = 79.52 N right Fy = 137.735 N down
To get the velocity of the jet as is leaves the vane.
ENTER: 60 + 960 SHIFT 2 3
(This is the resultant of 6 and 9 m/s )
Result:
13.07736.590
v = 13.077 m/s with angle with the x axis = 36.590

143
34. Curvilinear Translation
Example 72
In the figure, the 20 lb ball is forced to rotate around a
smooth inside surface of a conical shell at the rate of 1
revolution in π/4 sec. Find the tension in the cord and the
force on the conical shell. At what speed in rpm will the force
on the shell become zero. Prob 1130 page 303
Engineering Mechanics by Singer 2nd ed.
Solution:

20 lb

T N

300 300
m2r =
20/32.2(8)2(2) =
79.5 lb

ENTER: MODE 5 1
Input:
T N reversed Applied load
-sin 30 -cos 30 -79.5 x comp
cos 30 -sin 30 20 y comp

Result: T = 57.07 lb N = 58.85 lb

144
Force on the Shell = N
To get the speed when N = 0

20 lb

300

mr (2) = 20/32.2(2)2
= 1.24222

The unknowns now are T and 2


ENTER: MODE 5 1
Input:
T 2 Reversed Applied Load
-sin 30 1.2422 0 x comp
cos 30 0 20 y comp
Result:
T = 23.094 lb and 2 = 9.2956 rad/s
then  = 3.0489 rad /s
Convert this to RPM:
3.0489 rad /s x 60/(2π) = 29.11 RPM

145
Example 73
A body of weight W rests on a smooth inclined surface of the
frame shown. A peg attached to the frame forces the body to
rotate with it about the vertical axis. Determine the speed in
RPM at which the tension in the cord is equal to the weight of
the body.

m2 r =
1/32.2(w2)(5 cos 300 )
= 0.13448 2

Y
300 1
1
1

300
X

Solution:
Since no Weight is given, Assume that W = 1.
Then T = 1 (These two values are the applied loads.)
w and N are the unknowns.

ENTER: MODE 5 1

146
Input:
2 N Applied Loads
-0.13448 cos 30 0 1 sin 30 - 1
0.13448 sin 30 1 1cos 30
Result:
2 = 4.293 N = 0.5773 lb
 = 2.072 rad/s x 60 /(2π) = 19.79 RPM

35. Analysis of Projectile Motion


Example 74
A projectile is fired from the top of the cliff 300 ft high with a
vecocity of 1414 ft/s directed 450 with the horizontal. Find
the Range on the horizontal plane trough the base of the cliff.
Example 1101 page 294 Engineering Mechanics by Singer
2nd ed.
Solution:

147
The Working Equations for Projectile Motion are:
x = V0cos t
y = V0 sin t - ½ gt2
vy = v0 sin - gt
vx = v0 cos t
Use: y = V0 sin  t - ½ gt2
-300 = 1414 sin 45t – ½ (32.2) t2
MODE 5 3
Input:
-1/2(32.2) 1414 sin 45 300
Result: t = 62.4 and t = -0.2986
Then: t = 62.4
Use x = V0cos t
= 1414 cos 45 ( 62.4 )
= 62,390.58 ft

Alternative Solution:
y = x tan  - ½ g x2/(v02 cos2  )
y = - 300 x = ?  = 45 g = 32.2 and v0 = 1414 ft/s
ENTER: MODE 5 3
Input:
-1/2(32.2)÷(14142 cos(45)2) tan 45 300
Result: x = 62,391.607 and x = -298.5644
Answer: x = 62,391.607

148
Example 75 PAST CE BOARD
A man threw a piece of stone and hit a bird perched on the
top of a pole. The stone was thrown from P with a velocity of
24 m/s. If the pole was 14 m high and P is 1.5 m from the
ground and 6.7 m from the pole, at what angle did the stone
leave the mans hand. Neglect air resistance.
Solution:

Use
y = x tan  - ½ g x2/(v02 cos2  )
y = x tan  - ½ g x2/(v02) sec2 
or y = x tan  - ½ g(x2)/v02 ( 1 + tan2  )
y = 12.5 x = 6.7 v0 = 24 g = 9.81
Compute: - ½ g(x2)/v02
Result: -0.382266 Store this to A.
12.5 = 6.7 tan  + A ( 1 + tan2 )
ENTER: MODE 5 3
Input:
A 6.7 A – 12.5 =
Result: X1 = 15.3285 and X2 = 2.198
Then tan  = 15.3285  = 86.260
tan  = 2.198  = 65.5360

149
Example 76
A ball is thrown with an initial velocity of 30 m/s with angle
of 370.
a. What is the magnitude and direction of the velocity after 5
seconds.
b. What are the coordinates of the ball after 5 seconds?
c. What is the maximum Range of the ball.
d. What is the maximum height attained by thre ball.
Solution:

y
v

V= 30 m/s

300 x

a. From v = v0 + at
V = 30  30 + (9,81  (-90) ) 5
MODE 2 :
INPUT:
30  30 + (9,81  (-90) ) 5 SHIFT 2 3
Result: 42.83  (-52.660)
V = 42.83 m/s and  = -52.660

150
r = v0 t + ½ a t2
= ( 30  30 )5 + ½ ( 9.81  (-90) ( 52)
= 129.9038 – 47.625i
b. Then: x = 129.9038 and y = - 47.625

c. Since v = v0 +at
For maximum range: v = 30  (-30)
Thus: 30  (-30) = 30  30 + ( 9,81  (-90) t
t = 3.0581 s
Then r = vot + 1/2at2
=( 30  30 ) 3.0581 + ½ [( 9.81  (-90) ] (3.05812)
= 79.451
(Disregard the imaginary number
( Very small )
Also R = v02 sin 2  / g = 302 sin 60 / 9.81
= 79.451 m

d. For Maximum height:


velocity must have an angle of 00.
Since v = v0 + at
Then: v0 = 3030 + 9.81(-90)t
v0 + t( 9.8190 ) = 3030
Use MODE 5 1:
Input:
v t load
cos 0 9.81cos 90 30cos 30 Hor components
sin 0 9.81 sin 90 30sin 30 Ver components
Result: v = 25.981 and t = 1.529 s
Then r = v0t + ½ at2
= ( 3030 )( 1.529) + ½( 9.81 (-90) )(1.529)2
r = 39.724 + 11.467 i

151
Then max height = 11.467 m
Also max height = v02 sin2  / 2g
= 302 sin2 30 /( 2 x 9.81)
= 11.467 m

152
153

Das könnte Ihnen auch gefallen